Содержание

1.1.1. Строение электронных оболочек атомов химических элементов.

Химия — наука о веществах, их свойствах и превращениях.

Химическими веществами называют то, из чего состоит окружающий нас мир.

Свойства каждого химического вещества делятся на два типа: это химические, которые характеризуют его способность образовывать другие вещества, и физические, которые объективно наблюдаются и могут быть рассмотрены в отрыве от химических превращений. Так, например, физическими свойствами вещества являются его агрегатное состояние (твердое, жидкое или газообразное), теплопроводность, теплоемкость, растворимость в различных средах (вода, спирт и др.), плотность, цвет, вкус и т.д.

Превращения одних химических веществ в другие вещества называют химическими явлениями или химическими реакциями. Следует отметить, что существуют также и физические явления, которые, очевидно, сопровождаются изменением каких-либо физических свойств вещества без его превращения в другие вещества. К физическим явлениям, например, относятся плавление льда, замерзание или испарение воды и др.

О том, что в ходе какого-либо процесса имеет место химическое явление, можно сделать вывод, наблюдая характерные признаки химических реакций, такие как изменение цвета, образование осадка, выделение газа, выделение теплоты и (или) света.

Так, например, вывод о протекании химических реакций можно сделать, наблюдая:

— образование осадка при кипячении воды, называемого в быту накипью;

— выделение тепла и света при горении костра;

— изменение цвета среза свежего яблока на воздухе;

— образование газовых пузырьков при брожении теста и т.д.

Мельчайшие частицы вещества, которые в процессе химических реакций практически не претерпевают изменений, а лишь по-новому соединяются между собой, называются атомами.

Сама идея о существовании таких единиц материи возникла еще в древней Греции в умах античных философов, что собственно и объясняет происхождение термина «атом», поскольку «атомос» в буквальном переводе с греческого означает «неделимый».

Тем не менее, вопреки идее древнегреческих философов, атомы не являются абсолютным минимумом материи, т.е. сами имеют сложное строение.

Каждый атом состоит из так называемых субатомных частиц – протонов, нейтронов и электронов, обозначаемых соответственно символами p+, no и e. Надстрочный индекс в используемых обозначениях указывает на то, что протон имеет единичный положительный заряд, электрон – единичный отрицательный заряд, а нейтрон заряда не имеет.

Что касается качественного устройства атома, то у каждого атома все протоны и нейтроны сосредоточены в так называемом ядре, вокруг которого электроны образуют электронную оболочку.

Протон и нейтрон обладают практически одинаковыми массами, т.е. mp ≈ mn , а масса электрона почти в 2000 раз меньше массы каждого из них, т.е. mp/me ≈ mn/me ≈ 2000.

Поскольку фундаментальным свойством атома является его электронейтральность, а заряд одного электрона равен заряду одного протона, из этого можно сделать вывод о том, что количество электронов в любом атоме равно количеству протонов.

Так, например, в таблице ниже представлен возможный состав атомов:

 атом1атом2атом3атом4
ядро1p+1p+, 1n04p+, 3n04p+, 4n0
оболочка1e1e4e4e

Вид атомов с одинаковым зарядом ядер, т.е. с одинаковым числом протонов в их ядрах, называют химическим элементом. Таким образом, из таблицы выше можно сделать вывод о том, что атом1 и атом2 относятся в одному химическому элементу, а атом3 и атом4 — к другому химическому элементу.

Каждый химический элемент имеет свое название и индивидуальный символ, который читается определенным образом. Так, например, самый простой химический элемент, атомы которого содержат в ядре только один протон, имеет название «водород» и обозначается символом «Н», что читается как «аш», а химический элемент с зарядом ядра +7 (т.е. содержащий 7 протонов) — «азот», имеет символ «N» , который читается как «эн».

Как можно заметить из представленной выше таблицы, атомы одного химического элемента могут отличаться количеством нейтронов в ядрах.

Атомы, относящиеся к одному химическому элементу, но имеющие разное количество нейтронов и, как следствие массу, называют изотопами.

Так, например, химический элемент водород имеет три изотопа – 1Н, 2Н и 3Н. Индексы 1, 2 и 3 сверху от символа Н означают суммарное количество нейтронов и протонов. Т.е. зная, что водород – это химический элемент, который характеризуется тем, что в ядрах его атомов находится по одному протону, можно сделать вывод о том, что в изотопе 1Н вообще нет нейтронов (1-1=0), в изотопе 2Н – 1 нейтрон (2-1=1) и в изотопе 3Н – два нейтрона (3-1=2). Поскольку, как уже было сказано, нейтрон и протон имеют одинаковые массы, а масса электрона по сравнению с ними пренебрежимо мала, это значит, что изотоп 2Н практически в два раза тяжелее изотопа 1Н, а изотоп 3Н — и вовсе в три раза. В связи с таким большим разбросом масс изотопов водорода изотопам 2Н и 3Н даже были присвоены отдельные индивидуальные названия и символы, что не характерно больше ни для одного другого химического элемента. Изотопу 2Н дали название дейтерий и присвоили символ D, а изотопу 3Н дали название тритий и присвоили символ Т.

Если принять массу протона и нейтрона за единицу, а массой электрона пренебречь, фактически верхний левый индекс помимо суммарного количества протонов и нейтронов в атоме можно считать его массой, в связи с чем этот индекс называют массовым числом и обозначают символом А. Поскольку за заряд ядра любого атома отвечают протоны, а заряд каждого протона условно считается равным +1, количество протонов в ядре называют зарядовым числом (Z). Обозначив количество нейтронов в атоме буквой N, математически взаимосвязь между массовым числом, зарядовым числом и количеством нейтронов можно выразить как:

Согласно современным представлениям, электрон имеет двойственную (корпускулярно-волновую) природу. Он обладает свойствами как частицы, так и волны. Подобно частице, электрон имеет массу и заряд, но в то же время поток электронов, подобно волне, характеризуется способностью к дифракции.

Для описания состояния электрона в атоме используют представления квантовой механики, согласно которым электрон не имеет определенной траектории движения и может находиться в любой точке пространства, но с разной вероятностью.

Область пространства вокруг ядра, где наиболее вероятно нахождение электрона, называется атомной орбиталью.

Атомная орбиталь может обладать различной формой, размером и ориентацией. Также атомную орбиталь называют электронным облаком.

Графически одну атомную орбиталь принято обозначать в виде квадратной ячейки:

Квантовая механика имеет крайне сложный математический аппарат, поэтому в рамках школьного курса химии рассматриваются только лишь следствия квантово-механической теории.

Согласно этим следствиям, любую атомную орбиталь и находящийся на ней электрон полностью характеризуют 4 квантовых числа.

  • Главное квантовое число – n — определяет общую энергию электрона на данной орбитали. Диапазон значений главного квантового числа – все натуральные числа, т.е. n = 1,2,3,4, 5 и т.д.
  • Орбитальное квантовое число — l – характеризует форму атомной орбитали и может принимать любые целочисленные значения от 0 до n-1, где n, напомним, — это главное квантовое число.

Орбитали с l = 0 называют s-орбиталями. s-Орбитали имеют сферическую форму и не обладают направленностью в пространстве:

Орбитали с l = 1 называются p-орбиталями. Данные орбитали обладают формой трехмерной восьмерки, т.е. формой, полученной вращением восьмерки вокруг оси симметрии, и внешне напоминают гантель:

Орбитали с l = 2 называются d-орбиталями, а с l = 3 – f-орбиталями. Их строение намного более сложное.

3) Магнитное квантовое число – ml – определяет пространственную ориентацию конкретной атомной орбитали и выражает проекцию орбитального момента импульса на направление магнитного поля. Магнитное квантовое число ml соответствует ориентации орбитали относительно направления вектора напряженности внешнего магнитного поля и может принимать любые целочисленные значения от –l до +l, включая 0, т.е. общее количество возможных значений равно (2l+1). Так, например, при l = 0 ml = 0 (одно значение), при l = 1 ml = -1, 0, +1 (три значения), при l = 2 ml = -2, -1, 0, +1, +2 (пять значений магнитного квантового числа) и т.д.

Так, например, p-орбитали, т.е. орбитали с орбитальным квантовым числом l = 1, имеющие форму «трехмерной восьмерки», соответствуют трем значениям магнитного квантового числа (-1, 0, +1), что, в свою очередь, соответствует трем перпендикулярным друг другу направлениям в пространстве.

4) Спиновое квантовое число (или просто спин) — ms — условно можно считать отвечающим за направление вращения электрона в атоме, оно может принимать значения . Электроны с разными спинами обозначают вертикальными стрелками, направленными в разные стороны: ↓ и ↑.

Совокупность всех орбиталей в атоме, имеющих одно и то же значение главного квантового числа, называют энергетическим уровнем или электронной оболочкой. Любой произвольный энергетический уровень с некоторым номером n состоит из n2 орбиталей.

Множество орбиталей с одинаковыми значениями главного квантового числа и орбитального квантового числа представляет собой энергетический подуровень.

Каждый энергетический уровень, которому соответствует главное квантовое число n, содержит n подуровней. В свою очередь, каждый энергетический подуровень с орбитальным квантовым числом l, состоит из (2l+1) орбиталей. Таким образом, s-подуровень состоит из одной s-орбитали, p-подуровень – трех p-орбиталей, d-подуровень – пяти d-орбиталей, а f-подуровень — из семи f-орбиталей. Поскольку, как уже было сказано, одна атомная орбиталь часто обозначается одной квадратной ячейкой, то s-, p-, d- и f-подуровни можно графически изобразить следующим образом:

Каждой орбитали соответствует индивидуальный строго определенный набор трех квантовых чисел n, l и ml.

Распределение электронов по орбиталям называют электронной конфигурацией.

Заполнение атомных орбиталей электронами происходит в соответствии с тремя условиями:

  • Принцип минимума энергии: электроны заполняют орбитали, начиная с подуровня с наименьшей энергией. Последовательность подуровней в порядке увеличения их энергий выглядит следующим образом: 1s<2s<2p<3s<3p<4s≤3d<4p<5s≤4d<5p<6s…;

Для того чтобы проще запомнить данную последовательность заполнения электронных подуровней, весьма удобна следующая графическая иллюстрация:

  • Принцип Паули: на каждой орбитали может находиться не более двух электронов.

Если на орбитали находится один электрон, то он называется неспаренным, а если два, то их называют электронной парой.

  • Правило Хунда: наиболее устойчивое состояние атома является такое, при котором в пределах одного подуровня атом обладает максимально возможным числом неспаренных электронов. Такое наиболее устойчивое состояние атома называется основным состоянием.

Фактически вышесказанное означает то, что, например, размещение 1-го, 2-х, 3-х и 4-х электронов на трех орбиталях p-подуровня будет осуществляться следующим образом:

Заполнение атомных орбиталей от водорода, имеющего зарядовое число равное 1, до криптона (Kr) с зарядовым числом 36 будет осуществляться следующим образом:

Подобное изображение порядка заполнения атомных орбиталей называется энергетической диаграммой. Исходя из электронных диаграмм отдельных элементов, можно записать их так называемые электронные формулы (конфигурации). Так, например, элемент с 15ю протонами и, как следствие, 15ю электронами, т.е. фосфор (P), будет иметь следующий вид энергетической диаграммы:

При переводе в электронную формулу атома фосфора примет вид:

15P = 1s22s22p63s23p3

Цифрами нормального размера слева от символа подуровня показан номер энергетического уровня, а верхними индексами справа от символа подуровня показано количество электронов на соответствующем подуровне.

Ниже приведены электронные формул первых 36 элементов периодической системы Д.И. Менделеева.
период№ элементасимволназваниеэлектронная формула
I1Hводород1s1
2Heгелий1s2
II3Liлитий1s22s1
4Beбериллий1s22s2
5Bбор1s22s22p1
6Cуглерод1s22s22p2
7Nазот1s22s22p3
8Oкислород1s22s22p4
9Fфтор1s22s22p5
10Neнеон1s22s22p6
III11Naнатрий1s22s22p63s1
12Mgмагний1s22s22p63s2
13Alалюминий1s22s22p63s23p1
14Siкремний1s22s22p63s23p2
15Pфосфор1s22s22p63s23p3
16Sсера1s22s22p63s23p4
17Clхлор1s22s22p63s23p5
18Arаргон1s22s22p63s23p6
IV19Kкалий1s22s22p63s23p64s1
20Caкальций1s22s22p63s23p64s2
21Scскандий1s22s22p63s23p64s23d1
22Tiтитан1s22s22p63s23p64s23d2
23Vванадий1s22s22p63s23p64s23d3
24Crхром1s22s22p63s23p64s13d5 здесь наблюдается проскок одного электрона с s на d подуровень
25Mnмарганец1s22s22p63s23p64s23d5
26Feжелезо1s22s22p63s23p64s23d6
27Coкобальт1s22s22p63s23p64s23d7
28Niникель1s22s22p63s23p64s23d8
29Cuмедь1s22s22p63s23p64s13d10 здесь наблюдается проскок одного электрона с s на d подуровень
30Znцинк1s22s22p63s23p64s23d10
31Gaгаллий1s22s22p63s23p64s23d104p1
32Geгерманий1s22s22p63s23p64s23d104p2
33Asмышьяк1s22s22p63s23p64s23d104p3
34Seселен1s22s22p63s23p64s23d104p4
35Brбром1s22s22p63s23p64s23d104p5
36Krкриптон1s22s22p63s23p64s23d104p6

Как уже было сказано, в основном своем состоянии электроны в атомных орбиталях расположены согласно принципу наименьшей энергии. Тем не менее, при наличии пустых p-орбиталей в основном состоянии атома, нередко, при сообщении ему избыточной энергии атом можно перевести в так называемое возбужденное состояние. Так, например, атом бора в основном своем состоянии имеет электронную конфигурацию и энергетическую диаграмму следующего вида:

5B = 1s22s22p1

А в возбужденном состоянии (*), т.е. при сообщении некоторой энергии атому бора, его электронная конфигурация и энергетическая диаграмма будут выглядеть так:

5B* =  1s22s12p2

В зависимости от того, какой подуровень в атоме заполняется последним, химические элементы делят на s, p, d или f.

Нахождение s, p, d и f-элементов в таблице Д.И. Менделеева:

  • У s-элементов последний заполняемый s-подуровень. К данным элементам относятся элементы главных (слева в ячейке таблицы) подгрупп I и II групп.
  • У p-элементов заполняется p-подуровень. К p-элементам относят последние шесть элементов каждого периода, кроме первого и седьмого, а также элементы главных подгрупп III-VIII групп.
  • d-Элементы расположены между s – и p-элементами в больших периодах.
  • f-Элементы называют лантаноидами и актиноидами. Они вынесены вниз таблицы Д.И. Менделеева.

Электронные конфигурации простых ионов

Ионами называют частицы, имеющие либо положительный, либо отрицательный заряд. Ионы бывают простые и сложные. Простые ионы образованы одним химическим элементом, сложные – двумя или более элементами.

Положительно заряженные ионы называют катионами, отрицательно заряженные ионы – анионами. Заряд иона обозначают надстрочным индексом, сначала указывая величину заряда, затем его знак. При этом в случае, если заряд иона по модулю равен единице, пишут только знак (+ или -).

Примеры простых ионов: Na+, Ca2+, Al3+, S2-, I и т.д.
Примеры сложных ионов: SO42-, NO3, [AlF6]3- и т.д.

Рассмотрим детальнее простые ионы. Откуда у них может возникнуть заряд? Вспомним тот факт, что любой атом является электронейтральной частицей вследствие того, что количество отрицательно заряженных электронов в его оболочках равно количеству положительно заряженных протонов в его ядре.

Если мы «удалим» у атома часть электронов, то получим часть некомпенсированных положительных зарядов. При удалении части электронов у атома получим катион, при присоединении одного или нескольких электронов к атому получим анион.

Например, катион натрия Na+, от атома натрия Na0 отличает  то, что частица Na+ содержит в себе на один электрон меньше чем, атом натрия. Для того чтобы записать электронную конфигурацию катиона натрия Na+, сначала вспомним, сколько электронов имеет атом натрия. Сделать мы это можем, взглянув на порядковый номер химического элемента в таблице Д.И. Менделеева. Натрий имеет порядковый номер 11, следовательно, его атом содержит 11 электронов, тогда катион натрия будет иметь на один электрон меньше, то есть 10 электронов.
Далее распределим 10 электронов по энергетическим подуровням, исходя из всех тех же принципов, что использовались для записи электронных конфигураций атомов:

Аналогично попробуем записать электронно-графическую формулу сульфид-иона S2-. В таблице Д.И. Менделеева сера имеет порядковый номер, равный 16. Это значит, что атом серы S0 содержит 16 электронов. Отрицательный заряд иона серы равный 2- указывает на то, что у этого иона на два электрона больше, чем у атома серы, то есть 18 электронов. Тогда электронно-графическая формула и обычная электронная формула сульфид-иона S2- будут иметь вид:

Мы поняли, как записать электронно-графическую (или обычную электронную) формулу иона, зная точное его обозначение. Однако, для того чтобы успешно сдать ЕГЭ, нам нужно уметь самим определять заряды ионов, которые склонны образовывать те или иные химические элементы.  Причем уметь определять заряд наиболее устойчивых ионов мы обязаны только для элементов главных подгрупп.

Все очень просто, если учитывать, что атомы элементов главных подгрупп «хотят» получить электронную конфигурацию, как у ближайшего к ним по номеру в таблице атома благородного газа.

Например, определим, какой наиболее устойчивый ион образует магний. Смотрим в таблицу Д.И. Менделеева и видим, что ближайший по значению порядкового номера к нему благородный газ – неон. Атом неона содержит 10 электронов, значит и катион магния будет содержать 10 электронов, в то время как у обычного атома магния 12 электронов. Значит наиболее устойчивый катион магния будет иметь заряд 2+  (от 12 отнимаем 10), то есть мы можем обозначить его как Mg2+.

Установим формулу наиболее устойчивого иона хлора. Для этого снова смотрим в таблицу Д.И. Менделеева и видим, что ближайший по порядковому номеру благородный химический элемент — аргон. Атом аргона имеет 18 электронов, а атом хлора – 17 электронов. То есть наиболее устойчивый ион хлора содержит один «избыточный» электрон по сравнению с нейтральным атомом хлора. Таким образом, формулу наиболее устойчивого иона хлора можно записать как Cl.

Также есть еще один простой способ установления формул наиболее устойчивых ионов химических элементов, который заключается в том, чтобы попытаться найти их в ряду катионов и анионов таблицы растворимости. Если мы не находим анион соответствующего элемента, можно посмотреть на заряд его «родственника» по подгруппе. Например, в таблице растворимости мы не найдем ион кислорода, однако, в той же подгруппе, что и кислород, расположена сера, обозначение аниона которой мы легко находим в таблице растворимости – S2-. Следовательно, и наиболее устойчивый ион кислорода мы можем записать как O2-.

1.1.1 Строение электронных оболочек атомов элементов первых четырех периодов: s-, p- и d-элементы

Видеоурок: Строение атома: Строение электронных оболочек

Лекция: Строение электронных оболочек атомов элементов первых четырех периодов: s-, p- и d-элементы


Строение атома

 XX столетие является временем изобретения «модели строения атома». Исходя из предоставленного строения, удалось выработать следующую гипотезу: вокруг достаточно маленького по объему и размеру ядра, электроны совершают перемещения, схожие с перемещением планет вокруг Солнца. Последующее изучение атома показало, что сам атом и его строение гораздо сложнее, чем было установлено раньше. И в настоящее время, при огромных возможностях в научной сфере, атом исследован не до конца. Такие составляющие, как атом и молекулы, считаются предметами микромира. Поэтому данные части человек не способен рассмотреть самостоятельно. В этом мире установлены совершенно иные законы и правила, отличающиеся от макромира. Исходя из этого, исследование атома ведется на его модели.

Любому атому присвоен порядковый номер, закрепленный в Периодической таблице Менделеева Д.И. К примеру, порядковый номер атома фосфора (Р) — 15.

Итак, атом состоит из протонов (p+), нейтронов (n0) и электронов (e). Протоны и нейтроны образуют ядро атома, оно имеет положительный заряд. А электроны, совершающие перемещения вокруг ядра, «конструируют» электронную оболочку атома, имеющую отрицательный заряд. 

Сколько электронов в атоме? Это легко узнать. Достаточно посмотреть порядковый номер элемента в таблице. 

Так, число электронов фосфора равно 15. Количество электронов, содержащихся в оболочке атома, строго равно числу протонов, содержащихся в ядре. Значит и протонов в ядре атома фосфора 15

Масса протонов и нейтронов, составляющих массу ядра атома, одинакова. А электроны меньше в 2000 раз. Это означает что вся масса атома сосредоточена в ядре, массой электронов пренебрегают. Массу ядра атома мы также можем узнать из таблицы. Посмотрите изображение фосфора в таблице. Внизу мы видим обозначение 30, 974 – это и есть масса ядра фосфора, его атомная масса. При записи мы округляем эту цифру. Исходя из сказанного, запишем строение атома фосфора следующим образом:

 

(внизу слева написали заряд ядра – 15, вверху слева округленное значение массы атома – 31). 

Ядро атома фосфора:  


(внизу слева пишем заряд: протоны имеют заряд равный +1, а нейтроны не заряжены, то есть заряд 0; вверху слева масса протона и нейтрона, равная 1 – условная единица массы атома;  заряд ядра атома равен числу протонов в ядре, значит р=15, а число нейтронов нужно посчитать: из атомной массы вычесть заряд, т.е. 31 – 15 = 16). 

Электронная оболочка атома фосфора включает в себя

15 отрицательно заряженных электронов, уравновешивающих положительно заряженные протоны. Поэтому, атом – электронейтральная частица. 

Энергетические уровни
Рис.1

Далее нам необходимо подробно разобрать как распределяются электроны в атоме. Их движение не хаотично, а подчинено конкретному порядку. Какие — то из имеющихся электронов, притягиваются к ядру с достаточно большой силой, а другие наоборот, притягиваются слабо. Первопричина такого поведения электронов скрывается в разной степени удаленности электронов от ядра. То есть, ближе находящийся к ядру электрон, станет прочнее с ним взаимосвязан. Эти электроны просто нельзя отсоединить от электронной оболочки. Чем электрон дальше от ядра, тем проще «вытащить» его из оболочки. Так же, запас энергии электрона возрастает, по мере удаления от ядра атома. Энергия электрона определяется главным квантовым числом n, равняющимся любому натуральному числу (1,2,3,4…). Электроны, имеющие одинаковое значение n, образуют один электронный слой, как бы отгораживаясь от иных электронов, передвигающихся на удаленном расстоянии. На рисунке 1 изображены электронные слои, содержащиеся в электронной оболочке, в центре ядро атома.

Вы можете заметить, как по мере удаления от ядра увеличивается объем слоя. Следовательно, чем дальше слой от ядра, тем больше в нем электронов.  

Электронный слой, содержит в себе электроны, сходные по показателям энергии. Из – за этого, такие слои нередко именуют энергетическими уровнями. Сколько же уровней может содержать атом? Количество энергетических уровней равно номеру периода в таблице Менделеева Д.И. в котором находится элемент. К примеру, фосфор (Р) находится в третьем периоде, значит атом фосфора имеет три энергетических уровня. 

Рис. 2  

Как узнать максимальное количество электронов, располагающихся на одном электронном слое? Для этого используем формулу Nmax = 2n2, где n – это номер уровня. 

Получим, что первый уровень содержит всего 2 электрона, второй – 8, третий – 18, четвертый – 32.

Каждый энергетический уровень содержит в себе подуровни. Их буквенные обозначения: s-, p-, d- и f-. Посмотрите на рис. 2:

Здесь изображены подуровни. Е – это энергия (вспомните, что она возрастает по мере удаления электрона от ядра).

Разным цветом обозначены энергетические уровни, а полосками разной толщины подуровни. 

Самый тонкий подуровень обозначается буквой s. 1s – это s-подуровень первого уровня, 2s – это s-подуровень второго уровня и так далее. 

На втором энергетическом уровне появился p-подуровень, на третьем – d-подуровень, а на четвертом f-подуровень. 

Запомните увиденную закономерность: первый энергетический уровень включает одну s-подуровень, второй два s- и p- подуровня, третий три s-, p- и d-подуровня, а четвертый уровень четыре s-, p-, d- и f-подуровня. 

На s-подуровне могут находится только 2 электрона, на p-подуровне- максимум 6 электронов, на d-подуровне — 10 электронов, а на f-подуровне до 14 электронов.


Электронные орбитали

Область (место) где может находится электрон называется электронным облаком или орбиталью. Имейте ввиду, что говорится о вероятной области нахождении электрона, поскольку скорость его движения в сотни тысяч раз больше скорости движения иглы швейной машинки. Графически эта область изображается в виде ячейки:

       

В одной ячейке может находится два электрона. Судя по рисунку 2 можно сделать вывод о том, что s-подуровень, включающий не более двух электронов может содержать только одну s-орбиталь, обозначается одной ячейкой; p-подуровень имеет три р-орбитали (3 ячейки), d-подуровень пять d-орбиталей (5 ячеек), а f-подуровень семь f-орбиталей (7 ячеек).

Форма орбитали зависит от орбитального квантового числа (l — эль) атома. Атомный энергетический уровень, берет начало с s – орбитали, имеющей l = 0. Представленная орбиталь имеет сферическую форму. На уровнях, идущих после

s — орбитали, образуются p – орбитали с l = 1. P — орбитали напоминают форму гантели. Орбиталей, имеющих данную форму, всего три. Каждая возможная орбиталь содержит в себе не больше 2 – ух электронов. Далее располагаются более сложного строения d-орбитали (l = 2), а за ними f-орбитали (l = 3).

Рис. 3 Форма орбиталей
Электроны в орбиталях изображаются в виде стрелочек. Если орбитали содержат по одному электрону, то они однонаправленны – стрелкой вверх:

Если же в орбитали два электрона, то они имеют два направления: стрелкой вверх и стрелкой вниз, т.е. электроны разнонаправленны:

Такое строение электронов называется валентным.

Существуют три условия наполнения атомных орбиталей электронами:

  • 1 условие: Принцип минимального количества энергии. Заполнение орбиталей начинается с подуровня, имеющего минимальную энергию. Согласно данному принципу подуровни заполняются в таком порядке: 1s

    22s22p63s23p6 4s23d104p65s24d1066s25d14f14… Как мы видим, в некоторых случаях электрону энергетически выгоднее занять место в подуровне вышележащего уровня, хотя подуровень нижележащего уровня не заполнен. Например, валентная конфигурация атома фосфора выглядит так:

Рис. 4

  • 2 условие: Принцип Паули. Одна орбиталь включает 2 электрона (электронную пару) и не больше. Но возможно и содержание всего одного электрона. Его именуют неспаренным. 


  • 3 условие: Правило Хунда. Каждую орбиталь одного подуровня сначала заполняют по одному электрону, затем в них добавляются по второму электрону. В жизни мы видели аналогичную ситуацию, когда незнакомые пассажиры автобуса сначала занимают по одному все свободные сидения, а потом рассаживаются по два.

Электронная конфигурация атома в основном и возбужденном состоянии

Энергия атома, находящегося в основном состоянии, наименьшая. Если атомы начинают получать энергию из вне, к примеру, когда вещество нагревается, то они из основного состояния переходят в возбужденное. Этот переход возможен при наличии свободных орбиталей, на которые могут переместиться электроны. Но это временно, отдавая энергию, возбужденный атом возвращается в своё основное состояние.

Закрепим полученные знания на примере. Рассмотрим электронную конфигурацию, т.е. сосредоточение электронов по орбиталям атома фосфора в основном (невозбужденном состоянии). Еще раз обратимся к рис. 4. Итак, вспомним, что атом фосфора имеет три энергетических уровня, которые изображаются полудугами: +15)))  

Распределим, имеющиеся 15 электронов на эти три энергетических уровня:

Такие формулы называются электронными конфигурациями. Есть еще электронно – графические, они иллюстрируют размещение электронов внутри энергетических уровней. Электронно – графическая конфигурация фосфора выглядит так:

1s2 2s2 2p3s2 3p3 (здесь большие цифры – это номера энергетических уровней, буквы – это подуровни, а маленькие цифры – количество электронов подуровня, если их сложить, получится число 15).

В возбужденном состоянии атома фосфора 1 электрон переходит с 3s-орбитали на 3d-орбиталь, а конфигурация выглядит так: 1s2 2s2 2p3s1 3p33d1.

Строение электронных оболочек атомов элементов первых четырех периодов: s-, p- и d-элементы | by NikolayGolovko

Строение атома

XX столетие является временем изобретения “модели строения атома”. Исходя из предоставленного строения, удалось выработать следующую гипотезу: вокруг достаточно маленького по объему и размеру ядра, электроны совершают перемещения, схожие с перемещением планет вокруг Солнца. Последующее изучение атома показало, что сам атом и его строение гораздо сложнее, чем было установлено раньше. И в настоящее время, при огромных возможностях в научной сфере, атом исследован не до конца. Такие составляющие, как атом и молекулы, считаются предметами микромира. Поэтому данные части человек не способен рассмотреть самостоятельно. В этом мире установлены совершенно иные законы и правила, отличающиеся от макромира. Исходя из этого, исследование атома ведется на его модели.

Любому атому присвоен порядковый номер, закрепленный в Периодической таблице Менделеева Д.И. К примеру, порядковый номер атома фосфора (Р) — 15.

Итак, атом состоит из протонов (p+), нейтронов (n0) и электронов (e-). Протоны и нейтроны образуют ядро атома, оно имеет положительный заряд. А электроны, совершающие перемещения вокруг ядра, «конструируют» электронную оболочку атома, имеющую отрицательный заряд.

Сколько электронов в атоме? Это легко узнать. Достаточно посмотреть порядковый номер элемента в таблице.

Так, число электронов фосфора равно 15. Количество электронов, содержащихся в оболочке атома, строго равно числу протонов, содержащихся в ядре. Значит и протонов в ядре атома фосфора 15.

Масса протонов и нейтронов, составляющих массу ядра атома, одинакова. А электроны меньше в 2000 раз. Это означает что вся масса атома сосредоточена в ядре, массой электронов пренебрегают. Массу ядра атома мы также можем узнать из таблицы. Посмотрите изображение фосфора в таблице. Внизу мы видим обозначение 30, 974 — это и есть масса ядра фосфора, его атомная масса. При записи мы округляем эту цифру. Исходя из сказанного, запишем строение атома фосфора следующим образом:

(внизу слева написали заряд ядра — 15, вверху слева округленное значение массы атома — 31).

Ядро атома фосфора:

(внизу слева пишем заряд: протоны имеют заряд равный +1, а нейтроны не заряжены, то есть заряд 0; вверху слева масса протона и нейтрона, равная 1 — условная единица массы атома; заряд ядра атома равен числу протонов в ядре, значит р=15, а число нейтронов нужно посчитать: из атомной массы вычесть заряд, т.е. 31–15 = 16).

Электронная оболочка атома фосфора включает в себя 15 отрицательно заряженных электронов, уравновешивающих положительно заряженные протоны. Поэтому, атом — электронейтральная частица.

Энергетические уровни

Далее нам необходимо подробно разобрать как распределяются электроны в атоме. Их движение не хаотично, а подчинено конкретному порядку. Какие — то из имеющихся электронов, притягиваются к ядру с достаточно большой силой, а другие наоборот, притягиваются слабо. Первопричина такого поведения электронов скрывается в разной степени удаленности электронов от ядра. То есть, ближе находящийся к ядру электрон, станет прочнее с ним взаимосвязан. Эти электроны просто нельзя отсоединить от электронной оболочки. Чем электрон дальше от ядра, тем проще «вытащить» его из оболочки. Так же, запас энергии электрона возрастает, по мере удаления от ядра атома. Энергия электрона определяется главным квантовым числом n, равняющимся любому натуральному числу (1,2,3,4…). Электроны, имеющие одинаковое значение n, образуют один электронный слой, как бы отгораживаясь от иных электронов, передвигающихся на удаленном расстоянии. На рисунке 1 изображены электронные слои, содержащиеся в электронной оболочке, в центре ядро атома.

Вы можете заметить, как по мере удаления от ядра увеличивается объем слоя. Следовательно, чем дальше слой от ядра, тем больше в нем электронов.

Электронный слой, содержит в себе электроны, сходные по показателям энергии. Из — за этого, такие слои нередко именуют энергетическими уровнями. Сколько же уровней может содержать атом? Количество энергетических уровней равно номеру периода в таблице Менделеева Д.И. в котором находится элемент. К примеру, фосфор (Р) находится в третьем периоде, значит атом фосфора имеет три энергетических уровня.

Как узнать максимальное количество электронов, располагающихся на одном электронном слое? Для этого используем формулу Nmax = 2n2, где n — это номер уровня.

Получим, что первый уровень содержит всего 2 электрона, второй — 8, третий — 18, четвертый — 32.

Каждый энергетический уровень содержит в себе подуровни. Их буквенные обозначения: s-, p-, d- и f-. Посмотрите на рис. 2:

Здесь изображены подуровни. Е — это энергия (вспомните, что она возрастает по мере удаления электрона от ядра).

Разным цветом обозначены энергетические уровни, а полосками разной толщины подуровни.

Самый тонкий подуровень обозначается буквой s. 1s — это s-подуровень первого уровня, 2s — это s-подуровень второго уровня и так далее.

На втором энергетическом уровне появился p-подуровень, на третьем — d-подуровень, а на четвертом f-подуровень.

Запомните увиденную закономерность: первый энергетический уровень включает одну s-подуровень, второй два s- и p- подуровня, третий три s-, p- и d-подуровня, а четвертый уровень четыре s-, p-, d- и f-подуровня.

На s-подуровне могут находится только 2 электрона, на p-подуровне- максимум 6 электронов, на d-подуровне — 10 электронов, а на f-подуровне до 14 электронов.

Электронные орбитали

Область (место) где может находится электрон называется электронным облаком или орбиталью. Имейте ввиду, что говорится о вероятной области нахождении электрона, поскольку скорость его движения в сотни тысяч раз больше скорости движения иглы швейной машинки. Графически эта область изображается в виде ячейки:

В одной ячейке может находится два электрона. Судя по рисунку 2 можно сделать вывод о том, что s-подуровень, включающий не более двух электронов может содержать только одну s-орбиталь, обозначается одной ячейкой; p-подуровень имеет три р-орбитали (3 ячейки), d-подуровень пять d-орбиталей (5 ячеек), а f-подуровень семь f-орбиталей (7 ячеек).

Форма орбитали зависит от орбитального квантового числа (l — эль) атома. Атомный энергетический уровень, берет начало с s — орбитали, имеющей l = 0. Представленная орбиталь имеет сферическую форму. На уровнях, идущих после s — орбитали, образуются p– орбитали с l = 1. P — орбитали напоминают форму гантели. Орбиталей, имеющих данную форму, всего три. Каждая возможная орбиталь содержит в себе не больше 2 — ух электронов. Далее располагаются более сложного строения d-орбитали (l = 2), а за ними f-орбитали (l = 3).

Форма орбиталей

Электроны в орбиталях изображаются в виде стрелочек. Если орбитали содержат по одному электрону, то они однонаправленны — стрелкой вверх:

Если же в орбитали два электрона, то они имеют два направления: стрелкой вверх и стрелкой вниз, т.е. электроны разнонаправленны:

Такое строение электронов называется валентным.

Строение электронных оболочек атомов элементов первых четырех периодов: s-, р- и d-элементы. Электронная конфигурация атома. Основное и возбужденное состояние атомов — Современные представления о строении атома

Тесты с выбором ответа с решениями

1. Четыре энергетических уровня и шесть внешних электронов содержит атом

1) железа

2) хрома

3) селена

4) серы

2. Определите разность между числами нейтронов и электронов в атоме 56Fe.

1) 4

2) 26

3) 30

4) 56

3. Нейтральная частица, состоящая из положительно заряженного ядра и электронов, — это определение

1) молекулы

2) атома

3) изотопа

4) химического элемента

4. Суммарное число протонов, нейтронов и электронов в ионе Аl3+ равно

1) 40

2) 39

3) 38

4) 37

5. Максимальное число электронов на d-подуровне равно

1) 6

2) 8

3) 10

4) 12

6. Электронная формула атома железа

1) 1s22s22p63s23p63d64s2

2) 1s22s22p63s23p63d74s1

3) 1s22s22p63s23p63d84s0

4) 1s22s22p63s23p64s24p6

7. Укажите два элемента, каждый из которых имеет конфигурацию внешнего уровня 4s1.

1) К и Са

2) К и Na

3) Sc и Сu

4) Сr и Сu

8. Электронные формулы частиц S2- и S4+ — это соответственно

1) 1s22s22p63s23p6 и 1S22S22p6

2) 1s22s22p63s23p6 и 1s22s22p63s2

3) 1s22s22p63s2 И 1s22s22p63s23p6

4) 1s22s22p63s23p4 и 1s22s22p63s23p2

9. Электронная конфигурация 1s22s22p6 отвечает

1) атому неона и атому фтора

2) фторид-иону и атому натрия

3) атому неона и катиону лития

4) фторид-иону и катиону натрия

10. Напишите электронные формулы атома Сr и частицы Si4+. Ответ дайте в виде суммарного числа s-электронов в этих формулах.

1) 10

2) 11

3) 12

4) 13

Тесты с выбором ответа для самостоятельного решения

11. Атом 1737Э

1) содержит 20 нейтронов

2) содержит 37 протонов

3) содержит 20 электронов

4) имеет массу 54

12. Являются изотопами атомы

1) 919Э и 1019Э

2) 919Э и 1020Э

3) 919Э и 919Э

4) 919Э и 920Э

13. Электронная формула внешнего уровня 3s2Sp1 соответствует атому

1) бора

2) фосфора

3) азота

4) алюминия

14. Электронное строение атома элемента IIIA группы

1) 1s22s22p3

2) 1s22s22p63s23p1

3) 1S22S1

4) 1s22s22p63s1

15. Число неспаренных электронов в основном состоянии атома серы равно

1) 6

2) 4

3) 2

4) 0

16. Электронная формула иона О2-

1) 1s22s22p4

2) 1s22s22p2

3) 1s22s22p6

4) 1s22s22p8

17. Напишите электронные формулы атома Mg и иона Se2-. Ответ дайте в виде суммарного числа р-электронов в этих формулах.

1) 24

2) 22

3) 20

4) 18

18. Электронная конфигурация 1s22s22p63s23p4 соответствует частице

1) Сl

2) S2-

3) Сl+

4) Si4-

19. Два электрона на р-орбиталях внешнего электронного слоя в основном состоянии имеет атом

1) магния

2) кремния

3) титана

4) фосфора

20. Электронную конфигурацию инертного газа имеет ион

1) Zn2+

2) Fe2+

3) Cu2+

4) Са2+

Ответы на тесты раздела 1.1.1

1. 3 2. 1 3. 2 4. 4 5. 3 6. 1 7. 4 8. 2 9. 4 10. 2 11. 1 12. 4 13. 4 14. 2 15. 3 16. 3 17. 1 18. 3 19. 2 20. 4

Решения тестов раздела 1.1.1

Решение 1. Номер периода, в котором находится элемент, показывает число электронных уровней в атоме этого элемента. Так, у атомов железа, хрома и селена четыре уровня, а у атома серы — три. Номер группы, в которой находится элемент, показывает число валентных электронов. К валентным электронам относят s-электроны внешнего уровня для s-элементов, s- и р-электроны внешнего уровня для p-элементов. Для d-элементов валентными являются s-электроны внешнего уровня и d-электроны предвнешнего уровня. Так, у атома железа (3d64s2) есть 8 валентных электронов и 2 внешних электрона. У атома хрома (3d54s1) есть 6 валентных электронов и 1 внешний электрон. У атома селена (4s24p4) есть б валентных электронов и 6 внешних электронов.

Ответ: 3.

Решение 2. Элемент обозначают AzX, где X — символ элемента, Z — порядковый номер элемента в периодической таблице, А — массовое число. Число протонов в атоме равно Z. Число электронов в атоме равно Z. Число нейтронов равно разности А — Z. Порядковый номер железа в периодической таблице равен Z = 26. Число электронов в атоме железа равно числу протонов и равно 26, число нейтронов в атоме железа равно 56 — 26 = 30. Разность между числами нейтронов и электронов в атоме железа равна 30 — 26 = 4.

Ответ: 1.

Решение 3. Атом — нейтральная частица, состоящая из положительного ядра и электронов. Молекула — это группа атомов, связанных химическими связями, или молекула — это наименьшая частица вещества, сохраняющая его химические свойства. Химический элемент — вид атомов с одинаковым зарядом ядра. Изотопы — атомы одного элемента, имеющие разные массовые числа. Изотопы имеют одинаковые числа протонов и электронов и разные числа нейтронов. Например: атомы 1735Сl и 1737Сl.

Ответ 2.

Решение 4. Для атома алюминия порядковый номер Z = 13, а массовое число А = 27. Для атома 1327Аl имеем: число протонов 13, число нейтронов 27 — 13 = 14, число электронов 13. Для иона алюминия Аl3+ имеем число протонов 13, число нейтронов 14, число электронов 10. Суммарное число протонов, нейтронов и электронов в ионе Аl3+ равно 13 + 14 + 10 = 37.

Ответ: 4.

Решение 5. Подуровни состоят из орбиталей: s — из 1, р — из 3, d — из 5, а f — из 7 орбиталей. Согласно принципу Паули на орбитали не может быть больше двух электронов с разными спинами. Максимальные числа электронов на подуровнях равны: на s-подуровне — 2 электрона, на р-подуровне — б электронов, на d-подуровне — 10 электронов, на f-подуровне — 14 электронов.

Ответ: 3.

Решение 6. В электронной формуле подуровни записываются по уровням: 1s 2s2p 3s3p3d 4s4p…, а заполняются электронами по правилу Клечковского: 1s 2s2p 3s3p 4sSd 4p….

Атом железа содержит 26 электронов, которые распределяются по подуровням следующим образом: 1s22s22p63s23p63d64s2.

Ответ: 1.

Решение 7. Напишем электронные формулы атомов:

К (19ē) 1s22s22p63s23p64s1

Са (20ē) 1s22s22p63s23p64s2

Na (11ē) 1s22s22p63s1

Sc (21ē) 1s22s22p63s23p63d14s2

Для атомов первых четырех периодов из правила заполнения подуровней (правила Клечковского) есть два исключения — для атомов хрома и меди.

Для хрома Сr(24ē) вместо 1s22s22p63s23p63d44s2 имеем 1s22s22p63s23p63d54s1, а для меди Сu(29ē) вместо 1s22s22p63s23p63d94s2 имеем 1s22s22p63s23p63d104s1.

Ответ: 4.

Решение 8. Напишем электронные формулы атома и ионов серы:

S (16ē) 1s22s22p63s23p4

S2- (18ē) 1s22s22p63s23p6

S4+ (12ē) 1s22s22p63s2

S6+ (10ē) 1s22s22p6

Выбираем правильный ответ.

Ответ: 2.

Решение 9. Напишем электронные формулы атомов и ионов:

Ne (10ē) 1s22s22p6

F (10ē) 1S22S22P6

Na+ (10ē) 1s22s22p6

F (9ē) 1s22s22p5

Na (11ē) 1s22s22p63s1

Li+ (2ē) 1s2

Выбираем правильный ответ.

Ответ: 4.

Решение 10. Напишем электронные формулы. Для хрома Сr (24ē) 1s22s22p63s23p63d54s1 — исключение из правила заполнения подуровней — правила Клечковского. В атоме Сr — 7 s-электронов. Имеем: Si (14ē) 1s22s22p63s23p2 и Si4+ (10ē) 1s22s22p6. В ионе Si4+ — 4 s-электрона. Общее число s-электронов равно 7 + 4 = 11.

Ответ: 2.

Строение электронных оболочек атомов элементов первых четырех периодов

А1.

Строение электронных оболочек атомов элементов первых четырех периодов: s— и p—  и d— элементы. Электронная конфигурация атома. Основное и возбужденное состояние атомов

 

1. Количество электронов в атоме определяется

 


 

2 . Ион, в составе которого 16 протонов и 18 электронов, имеет заряд


1) +4       2) -2               3) +2                     4) -4

 

3. Внешний энергетический уровень атома элемента, образующего высший оксид состава ЭОз, имеет формулу

1) ns2np1     2) ns22         3) nз23           4) ns24

 

4. Конфигурация внешнего электронного слоя атома серы в невозбужденном состоянии

1) 4s2    2) 3s26           3)  3s24          4) 4s24

 

5. Электронную   конфигурацию   1s22s22p63s23p64s1   в  основном  состоянии имеет атом

1) лития

2) натрия

3) калия

4) кальция

 

6. Восьмиэлектронную внешнюю оболочку имеет ион

1) Р3+                      2) S2-                     3) С15+                    4) Fe2+

 

7. Двухэлектронную внешнюю оболочку имеет ион

1) S6+                      2) S2-                      3) Вг5+                    4) Sn4+

 

8. Число электронов в ионе железа Fe2+ равно

1) 54                      2) 28                      3) 58                      4) 24

 

9. Электронная конфигурация Is22s22p63s23p6 соответствует иону

1)  Sn2+                    2)  S2-                     3)  Cr3+                    4) Fe2+

 

10. В основном состоянии три неспаренных электрона имеет атом

1)   кремния

2)   фосфора

3)  серы

4)  хлора

 

11. Элемент с электронной конфигурацией внешнего уровня … 3s23p3 образует водородное соединение состава

1) ЭН4                   2) ЭН                    3) ЭН3                   4) ЭН2

 

12. Электронная конфигурация Is22s22p63s23p6 соответствует иону

1)  Сl                      2) N3-                    3) Br                       4) О2-

 

13. Электронная конфигурация Is22s22p6 соответствует иону

1) А13+                     2) Fe3+                     3) Zn2+                     4)  Cr3+

 

14. Одинаковую электронную конфигурацию внешнего уровня имеют Са2+ и

1) К+                     2) Аr                    3) Ва                     4) F

 

15. Атом   металла,   высший   оксид   которого   Ме2О3,   имеет   электронную формулу внешнего энергетического уровня

1) ns2пр1                  2) ns2пр2                     3) ns2np3                  4) ns2nps

 

16. Элемент,   которому   соответствует  высший   оксид  состава R2O7   имеет электронную конфигурацию внешнего уровня:

1) ns2np3          2)ns2np5             3) ns2np1       4) ns2np2

 

17.  Высший оксид состава R2O7 образует химический элемент, в атоме которого заполнение электронами энергетических уровней соответствует ряду чисел:

1) 2, 8, 1                  2) 2, 8, 7                  3) 2, 8, 8, 1            4) 2, 5

 

18.  У атома серы число электронов на внешнем энергетическом уровне и заряд ядра равны соответственно

1)4  и  + 16    2)6  и  + 32      3)6  и  + 16    4)4  и  + 32

 

19. Число валентных электронов у марганца равно

1) 1                      2) 3                      3) 5                     4) 7

 

20. Одинаковое электронное строение имеют частицы

1) Na0 и Na+         2) Na0 и  K0          3) Na+ и F             4) Cr2+ и Сr3+

 

21. Высший оксид состава ЭО3 образует элемент с электронной конфигурацией внешнего электронного слоя

1) ns2np1               2) ns2np3          3) ns2np4         4) ns2np6

 

22. Число энергетических слоев и число электронов во внешнем энергетическом слое атомов мышьяка равны соответственно

 


1)

4, 6

2)

2, 5

3)

3, 7

4)

4, 5

 

23 Иону Al3+ отвечает электронная конфигурация:

1) 1s22s22p6;        2) 1s22s22p63s1;       3) 1s22s22p63s23p    4) Is22s22p63s23p64s1  

 

24. Иону Zn2+ отвечает электронная конфигурация:

1) 1s22s22p63s23p63d84s2       2) 1s22s22p63s23p63d104s24p6      3 ) 1s22s22p63s23p63d10     4) Is22s22p63s23p64s1  

 

25. Химическому элементу соответствует летучее водородное соединение состава RH3. Электронная конфигурация внешнего уровня этого элемента

 


1)

3s23p1

2)

3s23p2

3)

3s23p3

4)

3s23p5

 

26. Атомы серы и кислорода имеют

 


1)

одинаковое число электронных слоев

2)

одинаковое число электронов внешнего электронного слоя

3)

одинаковое число протонов в ядре

4)

одинаковые радиусы

 

 

27.  Электронная конфигурация атома фтора

 


1)

1s22s22p5

2)

1s22s22p4

3)

1s22s22p6

4)

1s22s22p3

 

28. Сколько неспаренных электронов имеет атом углерода в состоянии sp3-гибридизации?

 


 

29.  У атома хлора на третьем электронном уровне имеется одна s-орбиталь, три p-орбитали и пять d-орбиталей. Максимальная валентность хлора равна

 


1)

четырем

2)

семи

3)

восьми

4)

девяти

 

 

30. Элемент, электронная конфигурация атома которого 1s22s22p63s23p2 образует водородное соединение

1) СН4           2) SiH4               3) H2O                  4) H2S

Достарыңызбен бөлісу:

s- и p- и d- элементы.

Строение электронных оболочек атомов элементов первых четырех периодов: s и p  и d элементы. Электронная конфигурация атома. Основное и возбужденное состояние атомов

 

1. Количество электронов в атоме определяется

 

числом протонов

2)

числом нейтронов

3)

числом энергетических уровней

4)

величиной относительной атомной массы

 

2 . Ион, в составе которого 16 протонов и 18 электронов, имеет заряд
1) +4       2) -2               3) +2                     4) -4

 

4. Конфигурация внешнего электронного слоя атома серы в невозбужденном состоянии

1) 4s2    2) 3s26           3)  3s24          4) 4s24

 

5. Электронную   конфигурацию   1s22s22p63s23p64s1   в  основном  состоянии имеет атом

1) лития 2) натрия 3) калия 4) кальция

6. Восьмиэлектронную внешнюю оболочку имеет ион

1) Р3+                      2) S2-                     3) С15+                    4) Fe2+

 

7. Двухэлектронную внешнюю оболочку имеет ион

1) S6+                      2) S2-                      3) Вг5+                    4) Sn4+

 

8. Число электронов в ионе железа Fe2+ равно

1) 54                      2) 28                      3) 58                      4) 24

 

9. Электронная конфигурация Is22s22p63s23p6 соответствует иону

1)  Sn2+                    2)  S2-                     3)  Cr3+                    4) Fe2+

 

10. В основном состоянии три неспаренных электрона имеет атом

1)   кремния 2)   фосфора 3)  серы 4)  хлора

 

11. Элемент с электронной конфигурацией внешнего уровня … 3s23p3 образует водородное соединение состава

1) ЭН4                   2) ЭН                    3) ЭН3                   4) ЭН2

 

12. Электронная конфигурация Is22s22p63s23p6 соответствует иону

1)  Сl                      2) N3-                    3) Br                       4) О2-

 

13. Электронная конфигурация Is22s22p6 соответствует иону

1) А13+                     2) Fe3+                     3) Zn2+                     4)  Cr3+

 

14. Одинаковую электронную конфигурацию внешнего уровня имеют Са2+ и

1) К+                     2) Аr                    3) Ва                     4) F

 

15. Атом   металла,   высший   оксид   которого   Ме2О3,   имеет   электронную формулу внешнего энергетического уровня

1) ns2пр1                  2) ns2пр2                     3) ns2np3                  4) ns2nps

 

16. Элемент,   которому   соответствует  высший   оксид  состава R2O7   имеет электронную конфигурацию внешнего уровня:

1) ns2np3          2)ns2np5             3) ns2np1       4) ns2np2

 

17.  Высший оксид состава R2O7 образует химический элемент, в атоме которого заполнение электронами энергетических уровней соответствует ряду чисел:

1) 2, 8, 1                  2) 2, 8, 7                  3) 2, 8, 8, 1            4) 2, 5

 

18.  У атома серы число электронов на внешнем энергетическом уровне и заряд ядра равны соответственно

1)4  и  + 16    2)6  и  + 32      3)6  и  + 16    4)4  и  + 32

 

19. Число валентных электронов у марганца равно

1) 1                      2) 3                      3) 5                     4) 7

 

20. Одинаковое электронное строение имеют частицы

1) Na0 и Na+         2) Na0 и  K0          3) Na+ и F             4) Cr2+ и Сr3+

 

21. Высший оксид состава ЭО3 образует элемент с электронной конфигурацией внешнего электронного слоя

1) ns2np1               2) ns2np3          3) ns2np4         4) ns2np6

 

22. Число энергетических слоев и число электронов во внешнем энергетическом слое атомов мышьяка равны соответственно

 

1)

4, 6

2)

2, 5

3)

3, 7

4)

4, 5

 

23 Иону Al3+ отвечает электронная конфигурация:

1) 1s22s22p6;        2) 1s22s22p63s1;       3) 1s22s22p63s23p1     4) Is22s22p63s23p64s1  

 

24. Иону Zn2+ отвечает электронная конфигурация:

1) 1s22s22p63s23p63d84s2       2) 1s22s22p63s23p63d104s24p6      3 ) 1s22s22p63s23p63d10     4) Is22s22p63s23p64s1  

 

25. Химическому элементу соответствует летучее водородное соединение состава RH3. Электронная конфигурация внешнего уровня этого элемента

 

23p1

2)

3s23p2

3)

3s23p3

4)

3s23p5

 

26. Атомы серы и кислорода имеют

 

 

27.  Электронная конфигурация атома фтора

 

22s22p5

2)

1s22s22p4

3)

1s22s22p6

4)

1s22s22p3

 

 

 

Ответы: 1-1, 2-2, 4-3,5-3,6-2,7-3,8-4,9-2, 10-2, 11-3, 12-1, 13-1, 14-1, 15-1, 16-2, 17-2, 18-3, 19-4, 20-3, 21-3, 22-4, 23-1, 24-3, 25-4, 26-2, 27-1

Абхазский государственный университет | Абхазский государственный университет

I. ТЕОРЕТИЧЕСКИЕ ОСНОВЫ ХИМИИ

Участник Олимпиады должен знать основы химической науки (факты, понятия, законы, теории), приводить примеры, устанавливать причинно-следственные связи, владеть химическим языком, называть вещества по принятой номенклатуре, знать формулы веществ.
Предмет и задачи химии. Место химии среди естественных наук.
Атомно-молекулярное учение. Молекулы. Атомы. Постоянство состава вещества. Относительная атомная и относительная молекулярная масса. Закон сохранения массы, его значение в химии. Моль — единица количества вещества. Молярная масса. Число Авогадро.

Химический элемент, простое вещество, сложное вещество. Знаки химических элементов и химические формулы. Расчет массовой доли химического элемента в веществе по его формуле.

Строение ядер атомов химических элементов и электронных оболочек атомов на примере элементов 1, 2, 3 и 4-го периодов периодической системы. Изотопы.
Периодический закон химических элементов Д.И.Менделеева. Распределение электронов в атомах элементов первых четырех периодов. Малые и большие периоды, группы и подгруппы. Характеристика отдельных химических элементов главных подгрупп на основании положения в периодической системе и строения атома. Значение периодического закона для понимания научной картины мира, развития науки и техники.
Типы химических связей: ковалентная (полярная и неполярная), ионная, водородная, металлическая. Примеры соединений со связями разных типов. Валентность и степень окисления.
Типы химических реакций: реакции соединения, разложения, замещения, обмена. Окислительно-восстановительные реакции. Тепловой эффект химических реакций.
Скорость химических реакций. Зависимость скорости от природы реагирующих веществ, концентрации, температуры. Катализ. Обратимость химических реакций. Химическое равновесие и условия его смещения.
Растворы. Растворимость веществ. Зависимость растворимости веществ от их природы, от температуры, давления. Тепловой эффект при растворении. Концентрация растворов. Значение растворов в промышленности, сельском хозяйстве, быту.
Электролитическая диссоциация. Степень диссоциации. Сильные и слабые электролиты. Реакции ионного обмена. Электролитическая диссоциация кислот, щелочей и солей. 

II. НЕОРГАНИЧЕСКАЯ ХИМИЯ

На основании периодического закона участники олимпиады долж­ны уметь давать сравнительную характеристику элементов по группам и периодам. Характеристика элемента включает элек­тронную конфигурацию атома; возможные валентности и сте­пени окисления элемента в соединениях; формы простых ве­ществ и основные типы соединений, их физические и химиче­ские свойства, лабораторные и промышленные способы полу­чения; распространенность элемента и его соединений в природе, практическое значение и области применения его соеди­нений. При описании химических свойств должны быть отражены реакции с участием неорганических и органических со­единений(кислотно-основные и окислительно-восстановительные превращения), а также качественные реакции. Химические свойства иллюстрируются уравнениями реакции в молекулярном и сокращенном ионном виде или электронными уравнениями с указанием окислителя и восстановителя, условиями проведения реакции.

Оксиды кислотные, основные, амфотерные. Способы получения и свойства оксидов. Амфотерность.
Основания, способы их получения и свойства. Щелочи, их получение, свойства и применение. Амфотерность.

Кислоты, свойства, способы получения. Реакция нейтрализации.
Соли. Состав и свойства. Гидролиз солей.

Водород. Химические, физические свойства. Взаимодействие с кислородом, оксидами металлов, с органическими веществами. Применение водорода как экологически чистого топлива и сырья для химической промышленности.

Кислород. Химические, физические свойства. Аллотропия. Применение кислорода. Круговорот кислорода в природе.

Вода. Физические и химические свойства. Кристаллогидраты. Значение воды в промышленности, сельском хозяйстве, быту, природе. Охрана водоемов от загрязнения.
Галогены. Общая характеристика галогенов. Соединения галогенов в природе, их применение.
Хлор. Физические, химические свойства. Реакции с неорганическими и органическими веществами. Получение хлора в промышленности. Соединения хлора. Применение хлора и его соединений.

Подгруппа углерода. Общая характеристика элементов IV группы главной подгруппы. Физические и химические свойства. Углерод, его аллотропные формы. Соединения углерода: оксиды (II, IV), угольная кислота и ее соли.

Кремний. Соединения кремния в природе, их использование      в         технике.

Подгруппа кислорода. Общая характеристика элементов главной подгруппы VI группы. Сера, ее физические и химические свойства. Соединения серы: сероводород, оксиды серы. Серная кислота, ее свойства, химические основы производства.

Общая характеристика элементов главной подгруппы V группы. Азот. Физические и химические свойства. Соединения азота: аммиак, соли аммония, оксиды азота, азотная кислота, соли азотной кислоты (физические и химические свойства). Производство аммиака. Применение аммиака, азотной кислоты и ее солей. Фосфор, его аллотропные формы, физические и химические свойства. Оксиды фосфора (V), фосфорная кислота и ее соли. Фосфорные удобрения.

Металлы. Положение в периодической системе. Особенности строения их атомов. Металлическая связь. Характерные физические и химические свойства. Коррозия металлов.
Щелочные металлы. Общая характеристика на основе положения в периодической системе Д.И. Менделеева. Соединения натрия, калия в природе, их применение. Калийные удобрения.
Общая характеристика элементов главных подгрупп II и III групп периодической системы Д.И. Менделеева. Кальций, его соединения в природе. Жесткость воды и способы ее устранения.

Алюминий. Характеристика алюминия и его соединений. Амфотерность оксида алюминия. Применение алюминия и его сплавов.

 

III. ОРГАНИЧЕСКАЯ ХИМИЯ

Характеристика каждого класса органических соединений включает особенности электронного и пространственного строения соединений данного класса, закономерности измене­ния физических и химических свойств в гомологическом ряду, номенклатуру, виды изомерии, основные типы химических ре­акций и их механизмы.

Характеристика конкретных соединений включает физи­ческие и химические свойства, лабораторные и промышленные способы получения, области применения. При описании химических свойств соединений необходимо учитывать реак­ции с участием, как радикала, так и функциональной группы. Участник олимпиады должен владеть основными понятиями органической химии, уметь иллюстрировать ответ уравнениями реакции с использованием структурных формул и обязательным указанием условий их протекания, называть органические вещества по международной номенклатуре.

Основные положения теории химического строения A.M. Бутлерова. Зависимость свойств веществ от химического строения. Изомерия. Электронная природа химических связей в молекулах органических соединений, способы разрыва связей, понятие о свободных радикалах.
Гомологический ряд предельных углеводородов (алканов), их электронное и пространственное строение, sp3-гибридизация. Метан. Номенклатура алканов, их физические и химические свойства. Циклопарафины. Предельные углеводороды в природе.

Этиленовые углеводороды (алкены). Гомологический ряд алкенов. Двойная связь, s- и р-связи, sр2-гибридизация. Физические свойства. Изомерия углеродного скелета и положение двойной связи. Номенклатура. Химические свойства. Получение углеводородов реакцией дегидрирования. Применение этиленовых углеводородов. Природный каучук, его строение и свойства.

Ацетилен. Тройная связь, sp-гибридизация. Гомологический ряд ацетилена. Физические и химические свойства, применение ацетилена. Получение его карбидным способом из метана.

Бензол, его электронное строение, химические свойства. Промышленное получение и применение бензола. Понятие о ядохимикатах, условиях их использования в сельском хозяйстве на основе требований охраны окружающей среды.

Взаимосвязь предельных, непредельных и ароматических углеводородов.
Природные источники углеводородов: нефть, природный и попутный нефтяные газы, уголь. Фракционная перегонка нефти. Крекинг. Ароматизация нефтепродуктов. Охрана окружающей среды при нефтепереработке.

Спирты, их строение, химические свойства. Изомерия. Номенклатура спиртов. Химические свойства спиртов. Применение метилового и этилового спиртов. Ядовитость спиртов, их губительное действие на организм человека. Генетическая связь между углеводородами и спиртами.

Фенол, строение, физические свойства. Химические свойства фенола. Применение фенола. Охрана окружающей среды от промышленных отходов, содержащих фенол.
Альдегиды, их строение, химические свойства. Получение и применение муравьиного и уксусного альдегидов.

Карбоновые кислоты. Гомологический ряд предельных одноосновных карбоновых кислот, их строение. Карбоксильная группа, взаимное влияние карбоксильной группы и углеводородного радикала. Физические и химические свойства карбоновых кислот.
Уксусная, пальмитиновая, стеариновая, олеиновая кислоты. Получение и применение карбоновых кислот.

Сложные эфиры. Строение, получение реакций этерификации. Химические свойства. Жиры в природе, их строение и свойства. Синтетические моющие средства, их значение. Защита окружающей среды от загрязнения синтетическими моющими средствами.
Глюкоза, ее строение, химические свойства, роль в природе. Сахароза, ее гидролиз.
Крахмал и целлюлоза, их строение, химические свойства, роль в природе. Применение целлюлозы и ее производных. Понятие об искусственных волокнах.
Амины как органические основания. Строение, аминогруппа. Взаимодействие аминов с водой и кислотами. Анилин, Получение анилина из нитробензола, практическое значение анилина. Аминокислоты. Строение, химические особенности, изомерия аминокислот. Аминокислоты, их значение в природе и применение. Синтез пептидов, их строение. Понятие об азотсодержащих гетероциклических соединениях на примере пиридина и пиррола.

Белки. Строение, структура и свойства белков. Успехи в изучении и синтезе белков. Значение микробиологической промышленности. Нуклеиновые кислоты, строение нуклеотидов. Принцип комплементарности в построении двойной спирали ДНК. Роль нуклеиновых кислот в жизнедеятельности клетки.

Общие понятия химии высокомолекулярных соединений: мономер, полимер, структурное звено, степень полимеризации, средняя молекулярная масса. Полимеризация, поликонденсация. Линейная, разветвленная структура полимеров. Зависимость свойств полимеров от их строения.

 

IV. ТИПЫ РАСЧЁТНЫХ ЗАДАЧ ПО ХИМИИ

Вычисление относительной молекулярной массы вещества по его формуле.

Вычисление массовых долей (процентного содержания) элементов в сложном веществе по его формуле.

Вычисление массовой доли растворённого вещества в рас­творе, если известна масса растворённого вещества и масса раствора.

Вычисление массы растворителя и массы растворённого вещества по известной массовой доле растворённого вещества и массе раствора.

Вычисление определённого количества вещества.

Вычисление количества вещества (в молях) по массе ве­щества.

Вычисление относительной плотности газообразных ве­ществ.

Вычисление объёма определённого количества газообраз­ного вещества при заданных условиях.

Вычисление массы газообразного вещества, занимающего определенный объем, при любых заданные значениях темпе­ратуры и давления.

Вычисление объема определенной массы газообразного вещества при любых заданных условиях.

Нахождение простейшей химической формулы вещества по массовым долям элементов.

Вычисление массы продукта реакции по известным мас­сам исходных веществ.

Вычисление массы продукта реакции по известным мас­сам одного из вступивших в реакцию веществ.

Вычисление выхода продукта реакции в процентах от тео­ретически возможного.

Вычисление массы (объема) продукта реакция по извест­ной массе (объёму) исходного вещества, содержащего опреде­лённую долю примесей.

Вычисление массовой доли компонентов смеси на основе данных задачи.

Установление молекулярной формулы газообразного ве­щества по продуктам сгорания.

Составление химических переходов (уравнений реакций) одних веществ в другие с использованием генетической связи между классами и соединениями (качественные задачи).

Экзаменационные билеты могут содержать как типовые, так и более сложные комбинированные задачи, состоящие из нескольких типов перечисленных видов расчетных задач. Кроме того, комбинированные задачи могут быть составлены по материалам различных разделов химии.

 

Рекомендуемая литература по химии

  1. Фельдман Ф.Г., Рудзитис Г.Е. Химия / Учебник по химии для 8-11 классов средней школы. – М., (разные годы издания).
  2. Хомченко И.Г. Сборник задач и упражнений по химии для средней школы. – М., (Различные издательства, разные годы).
  3. Хомченко Г.П. Пособие по химии для поступающих в вузы. – М., (Различные издательства, разные годы).
  4. Химия: Пособие – репетитор для поступающих в вузы / Под ред. А.С.Егорова. – Ростов н/Д., 2001.
  5. Кузьменко Н.Е., Еремин В.В., Попков В.А. Начала химии: В 2 т. – М., (разные годы издания).

 

 

 

Председатель предметной комиссии:                                             Ашхаруа Ф.Г.

 

 

Положение таблицы Менделеева и электронная конфигурация

Цель обучения
  • Используйте периодическую таблицу для определения свойств атомов, таких как группы и электронные конфигурации.

Ключевые моменты
    • Элементы организованы по периоду и группе, причем период соответствует основному уровню энергии, а группа — степени заполнения подоболочек.
    • Свойства атома напрямую связаны с количеством электронов на различных орбиталях, и периодическая таблица очень похожа на дорожную карту для этих орбиталей, так что химические свойства могут быть выведены по положению элемента на таблице.
    • Электроны во внешней или валентной оболочке особенно важны, потому что они могут участвовать в совместном использовании и обмене, который отвечает за химические реакции.

Условия
  • квантовое число Одно из определенных целых или полуцелых чисел, которые определяют состояние квантово-механической системы (например, электрона в атоме).
  • электронная оболочка: Коллективные состояния всех электронов в атоме, имеющие одно и то же главное квантовое число (визуализируемое как орбита, по которой движутся электроны).
  • орбитальная — спецификация энергии и плотности вероятности электрона в любой точке атома или молекулы.

Основные разделы периодической таблицы

Периодическая таблица представляет собой табличное отображение химических элементов, организованных на основе их атомных номеров, электронных конфигураций и химических свойств. Элементы представлены в возрастающем атомном номере. Основная часть таблицы представляет собой сетку 18 × 7. Элементы с одинаковым числом валентных электронов хранятся вместе в группах, таких как галогены и благородные газы.Есть четыре различных прямоугольных области или блока. F-блок обычно не включается в основную таблицу, а размещается ниже, поскольку встроенный f-блок часто делает таблицу непрактично широкой. Используя периодические тенденции, таблица Менделеева может помочь предсказать свойства различных элементов и отношения между свойствами. Таким образом, он обеспечивает полезную основу для анализа химического поведения и широко используется в химии и других науках.

Атомные орбитали

Электроны в частично заполненной самой внешней оболочке (или оболочках) определяют химические свойства атома; ее называют валентной оболочкой.Каждая оболочка состоит из одной или нескольких подоболочек, а каждая подоболочка состоит из одной или нескольких атомных орбиталей.

Свойства атома зависят в конечном итоге от числа электронов на различных орбиталях и от заряда ядра, который определяет компактность орбиталей. Чтобы связать свойства элементов с их расположением в периодической таблице, часто удобно использовать упрощенный вид атома, в котором ядро ​​окружено одной или несколькими концентрическими сферическими «оболочками», каждая из которых состоит из орбиталей с наивысшим главным квантовым числом, содержащих по крайней мере один электрон; это s- и p-орбитали и могут включать d- или f-орбитали, которые зависят от атома.Модель оболочки, как и любая научная модель, — это не столько описание мира, сколько упрощенный взгляд на него, который помогает нам понимать и соотносить различные явления.

Мы рассмотрим несколько визуализаций таблицы Менделеева. Однако сначала было бы поучительно посмотреть, как это построено с логической точки зрения. Таблица сегодня является результатом непрерывных усилий более чем 100 лет наблюдений, измерений, предсказаний и доказательств взаимосвязи химических и физических явлений с электронными конфигурациями и зарядами.

Периоды 1, 2 и 3

Начиная с простых элементов, первые три строки периодической таблицы, называемые периодами 1, 2 и 3, соответствуют уровням n = 1, n = 2 и n = 3.

Конфигурации электронных оболочек первых 18 элементов. Конфигурации электронных оболочек первых 18 элементов периодической таблицы. Соответствующие уровни энергии (n) указаны зелеными цифрами слева. Число электронов внешней оболочки представлено крайней правой цифрой в номерах групп.

Водород имеет 1 электрон на уровне 1s, а справа, гелий в Группе 18, имеет 2 электрона на уровне 1s, полностью заполненная оболочка, правило дуэта. Гелий — первый из благородных газов. Переходя к периоду 2, литий является первым элементом в строке с заполненной конфигурацией единиц. В течение периода сначала заполняются 2s, а затем 2p орбитали, достигая конфигурации для неона, следуя правилу октетов. Период 3 следует аналогичной схеме. Обратите внимание, что количество электронов внешней оболочки является основным фактором, определяющим валентность элемента.

Конфигурации электронных оболочек элементов Положение в периодической таблице на основе конфигурации электронных оболочек. На этом изображении показана вся таблица Менделеева с диаграммами атомов и электронных оболочек, заполняемых движением по таблице. На этом изображении численно показана электронная конфигурация, показывающая населенность электронов в каждой подоболочке, начиная каждый период с полностью заполненного благородного газа. Периодическая таблица с отображением электронных оболочек. Элементы в этой таблице расположены в стандартной конфигурации периодов и групп.Каждый блок включает представление структуры электронной оболочки элемента.

Показать источники

Boundless проверяет и курирует высококачественный контент с открытой лицензией из Интернета. Этот конкретный ресурс использовал следующие источники:

Электронные оболочки и орбитали

Обзор

Согласно модели Резерфорда-Бора , считалось, что электроны занимают фиксированные круговые орбиты вокруг ядра атома.Электроны с самыми низкими уровнями энергии занимали самые низкие орбиты. Электроны с более высокими уровнями энергии заняли бы более высокие орбиты. В то время как планетные орбиты в нашей Солнечной системе все лежат на двумерной орбитальной плоскости (или очень близко к ней), считается, что электронные орбиты занимают несколько различных орбитальных плоскостей , что породило концепцию трехмерного электрона. снаряды .

Теперь мы знаем, что модель Резерфорда-Бора неточно отражает поведение электронов.Они определенно не , а занимают аккуратные круговые орбиты — реальность намного сложнее. Тем не менее, модель Резерфорда-Бора все еще преподается в школах и колледжах, потому что она дает нам хорошую концептуальную основу для размышлений об электронах и их уровнях энергии. По этой причине мы начнем с изучения концепции электронных оболочек, а затем перейдем к изучению концепции орбиталей .

Прежде чем делать что-либо еще, давайте попробуем немного взглянуть на ситуацию.На момент написания периодическая таблица содержит сто восемнадцать (118) элементов. Первый элемент в таблице — это водород (H), у которого есть атомный номер , один (1), потому что он имеет один протон и один электрон. Последним элементом в таблице является оганессон (Og), который имеет атомный номер , сто восемнадцать (118), потому что он имеет сто восемнадцать протонов и сто восемнадцать электронов.

Таким образом, имея дело со сложностью электронной конфигурации атома, в худшем случае нужно учитывать сто восемнадцать электронов.Однако имейте в виду, что все элементы с номерами от 95 до 118 в периодической таблице являются синтетическими радиоизотопами. Они были произведены в лаборатории, не встречаются в природе и часто существуют очень недолго. Oganesson , например, имеет период полураспада всего семь десятых миллисекунды (0,7 мс).

Большинство элементов, с которыми физики и химики имеют дело в своей обычной работе, гораздо менее экзотичны.Тем не менее, предмет электронной конфигурации достаточно сложен, чтобы потребовать разумного количества исследований, если мы хотим получить хорошее представление о том, как все это работает. И важно достичь этого понимания, потому что способ, которым электроны организованы внутри атомов, определяет свойства различных элементов и то, как они взаимодействуют друг с другом.

Чтобы отвлечься на мгновение, интересно подумать, что все, что вы есть, и все, что вы можете видеть, слышать или трогать — фактически, весь мир вокруг вас — все это состоит из менее чем сотни различных видов атомных частиц. структурный элемент!

Электронные оболочки в модели Резерфорда-Бора

В модели атома Резерфорда-Бора электроны занимают электронные оболочки, каждая из которых расположена на определенном расстоянии от ядра.Каждый из электронов в определенной электронной оболочке обладает дискретным количеством энергии, обозначенным квантовым числом ( n ). Те, у кого меньше всего энергии, находятся в ближайшей к ядру электронной оболочке. Электроны в оболочке, наиболее удаленной от ядра (называемой валентной оболочкой ), обладают наибольшим количеством энергии.

Из ста восемнадцати известных элементов электронные конфигурации первых ста восьми до сих пор определены.Предлагаемые конфигурации электронов для остальных элементов основаны на том, что мы ожидаем увидеть, а не на фактических данных наблюдений (эти элементы имеют чрезвычайно короткие периоды полураспада, что затрудняет получение точных измерений).

Электронная конфигурация известных элементов представлена ​​с помощью семи основных электронных оболочек, каждая из которых представлена ​​концентрическим кругом с ядром в центре, как показано ниже.Оболочки обозначены квантовыми числами от n = 1 до n = 7, причем n = 1 — ближайшая к ядру оболочка (вы также можете увидеть их, обозначенные заглавными буквами K, L, M, N , O, P и Q, где K — ближайшая к ядру оболочка).


Элемент Калий (Hs) имеет 108 электронов в 7 электронных оболочках.


Электроны и таблица Менделеева

Периодическая таблица упорядочивает элементы в соответствии с количеством протонов в ядре каждого элемента.Поскольку в неионизированном атоме количество электронов будет таким же, как и количество протонов, мы можем вывести количество электронов в атоме конкретного элемента из положения этого элемента в периодической таблице (то есть его атомного номера ).

Атомы имеют тенденцию принимать наиболее стабильную конфигурацию и минимально возможный уровень энергии. По этой причине электронные оболочки в атоме (обычно) заполняются в строгом порядке, начиная с самого низкого энергетического уровня.Каждая из электронных оболочек (от n 1 до n 7 или от K до Q) может содержать максимальное количество электронов, определяемое формулой: электронная емкость = 2 n 2 (где n номер электронной оболочки), как показано ниже. Обратите внимание, однако, что максимальное количество электронов, реально наблюдаемых в электронной оболочке на сегодняшний день, составляет тридцать два (32).

Электронная оболочка 2 n 2
n 1 (K) 2
n 2 (L) 8 n 3 (M) 18
n 4 (N) 32
n 5 (O) 50
n 6 (P) 72
n 7 (Q) 98

Положение элемента в периодической таблице многое говорит нам о его электронной конфигурации.Сама таблица делится на строки ( или периоды ) и столбцы (или групп ). В таблице семь периодов, пронумерованных от 1 до 7. Элементы в каждом периоде имеют такое же количество электронных оболочек, как и номер этого периода. Водород и гелий находятся в периоде 1, и оба имеют одну электронную оболочку. Углерод, азот и кислород находятся в периоде 2, и все они имеют две электронные оболочки. И так далее.

В периодической таблице восемнадцать групп .Элементы одной группы обычно имеют схожие химические свойства. Это связано с тем фактом, что элементы в одной группе имеют тенденцию иметь одинаковое количество электронов на своей внешней электронной оболочке (валентной оболочке). Элементы в группах 1, 2, 13, 14, 15, 16, 17 и 18 имеют 1, 2, 3, 4, 5, 6, 7 и 8 электронов в их валентных оболочках соответственно.

Группы с 3 по 12 содержат три особые группы металлов, называемые переходными металлами , лантаноидами и актиноидами .Мы рассмотрим значение этих особых групп в другом месте. Для целей этого обсуждения важно отметить, что большинство из шестидесяти восьми элементов в этих десяти группах имеют двух электронов на своей внешней оболочке. Небольшое количество элементов имеет только , один, , электрон на своей внешней оболочке, а один элемент (лоуренсий) имеет , три, .


В периодической таблице элементы разбиты на периодов (строки 1-7) и групп (столбцы 1-18).


Более подробные версии периодической таблицы (вы можете найти отличный пример здесь) часто показывают электронную конфигурацию в виде списка значений, разделенных запятыми, с указанием количества электронов в каждой оболочке.Например, кремний (Si) будет иметь электронную конфигурацию 2, 8, 4. Электронные оболочки 1 n и 2 n заполнены, содержащие , два, и , восемь электронов соответственно, а электронная оболочка 3 n содержит четыре электронов, всего четырнадцать электронов.

Ранее мы говорили, что электронные оболочки в атоме обычно заполняются в строгом порядке, начиная с самого низкого энергетического уровня.Это явно относится к элементам первых трех периодов в периодической таблице. После этого все становится немного сложнее. В четвертом периоде, например, мы видим, что электроны появляются в оболочке 4 n до того, как оболочка 3 n полностью заполнится. Мы надеемся, что причины этой очевидной аномалии станут ясны со временем, когда мы начнем смотреть на орбиталей .

На рисунке ниже показаны первые десять элементов периодической таблицы.Обратите внимание, что гелий и неон имеют полные внешние оболочки . Оба эти элемента можно найти в 18-й группе периодической таблицы. Все элементы группы 18 (кроме гелия) имеют восемь электронов в валентной оболочке. Это делает их очень стабильными и нереактивными. Элементы Группы 18 гелий , неон , аргон , криптон , ксенон и радон , когда-то известные как инертные газы , сегодня чаще называются благородными газами , потому что они, как правило, не взаимодействуют с другими элементами.


Первые десять элементов периодической таблицы


Электронная конфигурация элемента и, в частности, количество электронов, которые он имеет в своей валентной (внешней) оболочке, имеет огромное влияние на то, как атомы этого элемента взаимодействуют с атомами других элементов. Например, элементам Группы 17 фтор , хлор , бром и йод (вместе известные как галогены ) всем нужен один дополнительный электрон для достижения полной валентной оболочки.

Эти элементы активно взаимодействуют с так называемыми щелочными металлами ( литий , натрия , калий , рубидий и цезий ) в группе 1 периодической таблицы, все из которых имеют только и . электрон в их валентных оболочках. Когда натрий реагирует с хлором, эти элементы образуют высокостабильное ионное соединение , хлорид натрия (NaCl), более известное как поваренная соль.Комбинируя таким образом, оба элемента эффективно образуют законченные внешние оболочки.

Орбитали

Теперь мы, конечно, знаем, что электроны не движутся вокруг ядра атома по аккуратным круговым орбитам, описываемым моделью Резерфорда-Бора. Фактически, благодаря работе Гейзенберга и Шредингера мы знаем, что в любой момент времени электрон может находиться где угодно внутри электронного облака.Вот тут-то и пригодится орбиталей .

Мы заявляли, что мы не можем знать с какой-либо уверенностью, где электрон будет в любой момент времени. Однако мы можем идентифицировать трехмерную область в пространстве, занимаемом атомом, где конкретный электрон можно найти в течение девяноста процентов (90%) времени. Мы называем эту область орбитой . Название вводит в заблуждение, поскольку орбиталь электрона не имеет ничего общего с «орбитой».

Орбитали могут иметь разную форму, а геометрия может быть довольно сложной. Самый простой вид орбитали — это орбиталь s , которая имеет сферическую форму с ядром атома в центре. Другие орбитали включают орбитали p , d и f . По историческим причинам обозначенные буквы обозначают резкий , основной , диффузный и основной , на основе наблюдений спектральных линий, связанных с ними, но эта связь больше не имеет никакого значения.Орбиты за пределами орбиталей f обозначаются как g , h , i и так далее (то есть в алфавитном порядке).

Орбиталь p имеет форму гантели, опять же с центром в ядре атома. Орбитали такого типа могут сосуществовать внутри электронной оболочки, но каждая выровнена по другой перпендикулярной оси (представьте ось x , y или z , проходящую через ядро ​​атома).Непрактично пытаться описать геометрию всех различных типов электронных орбиталей, но на следующем рисунке показаны некоторые из возможных перестановок.

Каждая из орбиталей, показанных на рисунке выше, даже те, которые выглядят довольно сложными, могут содержать максимум двух электронов. Таблица Менделеева теперь содержит более сотни элементов, так что вы, вероятно, можете представить, насколько сложной может стать геометрия орбиты! К счастью, на экзамене обычно не требуется описывать что-либо, слишком продвинутых в терминах орбитальной геометрии.Возможно, более важно понять, как орбитали связаны с электронными оболочками.

Хотя мы знаем, что электронные оболочки на самом деле не состоят из электронов, движущихся по аккуратным круговым орбитам вокруг ядра, они достаточно реальны в том смысле, что каждая электронная оболочка (иногда называемая основным энергетическим уровнем ) представляет собой определенный квантовый энергетический уровень, представлен главным квантовым числом ( n ).Каждая электронная оболочка состоит из одной или нескольких подоболочек , а каждая подоболочка состоит из одной или нескольких орбиталей .

Часто утверждают, что электроны в одной и той же электронной оболочке имеют одинаковые уровни энергии. В лучшем случае это обобщение. Все электроны в одной подоболочке будут иметь точно такой же уровень энергии. Тем не менее, в то время как общее количество уровней энергии в электронных оболочках увеличивается с главным квантовым числом n , уровни энергии между подоболочками в одной и той же электронной оболочке будут разными.Электроны в подоболочках, которые заполняются позже, будут иметь больше энергии, чем электроны в подоболочках, которые заполняются раньше. В некоторых случаях, как мы увидим, диапазоны энергий, связанные с двумя электронными оболочками , перекрываются .

Обратите внимание, что орбитали не только обозначаются обозначенной буквой (s, p, d, f и т. Д.), Но и могут быть связаны с конкретной электронной оболочкой с помощью соответствующего номера. Например, орбитали s в первых трех электронных оболочках будут называться орбиталями 1 s , 2 s и 3 s .Аналогично, орбитали p в электронных оболочках 2 n , 3 n и 4 n будут обозначаться как орбитали 2 p , 3 p и 4 p соответственно.

Вторичные, магнитные и спиновые квантовые числа

Прежде чем двигаться дальше, мы хотели бы ввести некоторые дополнительные квантовые числа. Первым из них является вторичное квантовое число (или угловой момент , или азимутальный ) квантовое число & ell; .Этот номер обозначает форму орбиты . Обычно для этой цели мы используем буквы s , p , d , f и т. Д., Чтобы избежать путаницы с главным квантовым числом ( n ). Значения & ell; относятся к орбитальным типам следующим образом:

Вторичное квантовое число ( & ell; ) Орбитальный тип
0 s
1 p
3 f
4 g
5 h
.. .. . .

Вторичное квантовое число идентифицирует подоболочку, поскольку каждая подоболочка состоит из набора орбиталей определенного типа внутри оболочки. Это также имеет значение в отношении уровня энергии подоболочки, который немного увеличится до & ell; увеличивается.

Следующее квантовое число, о котором мы хотим поговорить, — это магнитное квантовое число , m .Этот номер обозначает ориентацию орбитали в космосе. Все орбитали подоболочки будут иметь одинаковые значения энергии, но разную пространственную ориентацию. Для любой подоболочки значения, которые может принимать м , будут целыми числами в диапазоне — & ell; . . . 0. . . & ell; . Таким образом, количество орбиталей в любой подоболочке будет 2 & ell; + 1.

Последним квантовым числом, представляющим интерес в этом контексте, является квантовое число спина , s .Это число дает собственный угловой момент (или спиновый угловой момент или просто спин ) электрона и обозначается буквой s (термин собственный угловой момент просто означает, что электрон вращается. вокруг собственной оси).

По причинам, которые выходят за рамки этой страницы (и потому что прямое объяснение ускользало даже от физиков, лауреатов Нобелевской премии), квантовое число спина ( s ) электрона может принимать значения s = + 1 / 2 или с = — 1 / 2 .Эти две ориентации вращения иногда называют с вращением вверх и с вращением вниз соответственно. Здесь важно отметить, что, когда два электрона занимают одну и ту же орбиталь, они должны иметь противоположных ориентаций спина .


Электрон имеет квантовое число спина — 1 / 2 или + 1 / 2


В 1925 году немецкий физик Фридрих Герман Хунд (1896–1997) установил набор правил, управляющих электронной конфигурацией в атомах.Тот, который применяется здесь (и тот, который обычно называют правилом Хунда ), заключается в том, что, если доступны две или более орбиталей в одной подоболочке, электроны сначала займут пустую орбиталь, если таковая имеется. Только после того, как все орбитали подоболочки будут иметь по одному электрону, электроны начнут спариваться. Первый электрон на орбите имеет спин + 1 / 2 , а второй имеет спин — 1 / 2 .

Четыре квантовых числа (главное квантовое число n , вторичное квантовое число & ell; , магнитное квантовое число m и спиновое квантовое число s ) вместе описывают уникальное квантовое состояние одиночного электрона.Они полностью описывают движение и траектории каждого электрона в атоме.

Никакие два электрона не могут иметь одну и ту же комбинацию квантовых чисел (принцип, известный как Принцип исключения Паули , названный в честь австрийского физика Вольфганга Эрнста Паули (1900-1958), который открыл его и внес значительный вклад в эту область). спиновой теории (исследование собственного углового момента элементарных частиц).

Квантовые числа важны по ряду причин. Их можно использовать для определения электронной конфигурации атома и вероятного расположения электронов внутри атома. Они также являются факторами, определяющими такие характеристики, как энергия ионизации и атомный радиус атома, то, как атом будет вести себя в магнитном поле, и может ли атом генерировать собственное магнитное поле.

Узлы

Большинство орбиталей имеют один или несколько узлов .Узел — это точка, в которой вероятность появления электрона равна нулю. Есть два типа узлов — радиальные узлы и угловые узлы. Радиальный узел — это, по сути, набор точек, которые лежат на некотором заданном радиальном расстоянии от ядра. Угловой узел обычно представлен как плоскость симметрии, разделяющая орбиталь пополам.

Для всех типов орбиталей количество узлов увеличивается с увеличением главного квантового числа ( n ).Есть также несколько простых формул, которые вы можете использовать, если вам нужно определить количество и тип узлов для конкретной орбиты. Это следующие:

количество узлов = n — 1
количество угловых узлов = & ell;
количество радиальных узлов = n & ell; — 1

где n — главное квантовое число, а & ell; — вторичное квантовое число.Давайте посмотрим на пару примеров, чтобы увидеть, как это работает. Предположим, мы хотим определить количество и тип узлов на орбитали типа p в электронной оболочке 3 n . Мы знаем, что значение n равно 3 (дано), и мы видели ранее, что значение & ell; для орбиты типа p равно 1. Таким образом, мы имеем:

количество узлов = n — 1 = 3 — 1 = 2
количество угловых узлов = & ell; = 1
количество радиальных узлов = n & ell; — 1 = 3 — 1 — 1 = 1

Теперь предположим, что мы хотим найти количество и тип узлов на орбитали типа d в электронной оболочке 5 n .Цифры становятся все больше, но это все еще довольно тривиальная проблема. Значение n равно 5, а значение & ell; для орбиты типа d равно 2. Таким образом, мы имеем:

количество узлов = n — 1 = 5 — 1 = 4
количество угловых узлов = & ell; = 2
количество радиальных узлов = n & ell; — 1 = 5 — 2 — 1 = 2

Орбитали

s

Неудивительно, что простейшую электронную конфигурацию можно увидеть в атоме водорода, который имеет всего один электрон.Таким образом, атом водорода имеет одноэлектронную оболочку , состоящую из одиночной подоболочки , содержащей одиночную s-орбиталь . Орбитали s являются сферическими и являются единственными орбиталями, которые встречаются в каждой электронной оболочке. Как и все орбитали, орбитали s имеют номер, связанный с ними; главное квантовое число ( n ) оболочки, в которой они появляются. На рисунке ниже показаны первые три орбитали s .


Орбитали 1 с , 2 с и 3 с


Орбитали s имеют только радиальных узлов (орбиталь 1 s вообще не имеет узлов — это просто сфера электронной плотности). Надеюсь, следующая иллюстрация поможет вам визуализировать распределение электронов на орбиталях s .


Распределение электронов на орбиталях 1 с , 2 с и 3 с


Для всех типов орбиталей — как мы упоминали ранее — количество узлов увеличивается с увеличением главного квантового числа ( n ). Орбитали s не являются исключением — орбиталь 2 s имеет один радиальный узел, орбиталь 3 s имеет два радиальных узла и так далее (обратите внимание, что электронная оболочка может содержать только одну орбиталь s ).

Орбитали

p

Электронная оболочка 1 n может содержать только два электрона, каждый из которых будет занимать одну орбиталь s . С другой стороны, электронная оболочка 2 n может содержать до восьми электронов. Из этих восьми электронов первые два займут орбиталь 2 с . Любые дополнительные электроны займут одну из трех орбиталей p .На рисунке ниже показаны все орбитали, обнаруженные в электронных оболочках 1 n и 2 n .

Как вы можете видеть, три орбитали p в форме гантели лежат под прямым углом друг к другу вдоль воображаемых осей x , y и z . Мы идентифицируем отдельные орбитали p в электронной оболочке с помощью меток p x , p y и p z .


Электронная оболочка 2 n является первой, содержащей p орбиталей.


Как мы видели, значение вторичного квантового числа ( & ell; ) для орбиталей p равно единицам (1). Это означает, что каждая орбиталь p имеет один угловой узел . Вы можете представить угловой узел как двумерную плоскость, разделяющую ядро ​​пополам, с одной долей орбиты p по обе стороны от нее.В качестве примера, угловой узел для орбиты p x (это тот, который лежит вдоль оси x ) будет плоскостью, на которой лежат оси y и z .

Поскольку вероятность нахождения электрона в узле равна нулю, вам может быть интересно, как электронам удается попасть из одного лепестка орбитали p в другой. Хотя подробное объяснение, возможно, выходит за рамки этой страницы, достаточно сказать, что оно связано с тем фактом, что электроны могут вести себя как частицы, так и волны.

Возможные значения магнитного квантового числа m для орбитали p равны -1, 0 и +1 (поскольку & ell; равно единице), что означает, что может быть три орбитали p в любой из электронные оболочки кроме 1 n. Как только орбиталь s в каждой электронной оболочке будет иметь дополнение из двух электронов, следующие шесть электронов найдут дом на одной из орбиталей p .

Орбитали

d

Для первых восемнадцати элементов периодической таблицы до аргона включительно (т.е. первых трех периодов таблицы) все электроны будут найдены либо на орбиталях s , либо на орбиталях p . Фактически, это продолжает иметь место для первых двух элементов четвертого периода — калия и кальция .В этих двух элементах мы видим электроны, появляющиеся на орбитали s электронной оболочки 4 n , несмотря на то, что электронная оболочка 3 n еще не заполнена.

Причина этой кажущейся аномалии заключается в том, что уровень энергии орбитали s в электронной оболочке 4 n (обозначенной как орбиталь 4 s ) на самом деле немного ниже уровня энергии орбиталей d в электроне. shell 3 n , и мы уже видели, что электроны (обычно) сначала занимают орбитали с самыми низкими уровнями энергии (чтобы усложнить ситуацию, уровень энергии орбитали 4 s увеличивается, когда 3 d Орбитали начинают заполняться, но мы не будем вдаваться в подробности).

Элемент , скандий, — один из так называемых переходных металлов , — следует за кальцием в периодической таблице. Скандий имеет девять (9) электронов в электронной оболочке 3 n . Два из них размещены на орбите 3 s и еще шесть — на орбите 3 p . Девятый электрон занимает орбиталь d . Здесь все становится по-настоящему интересным.

Орбитали d имеют вторичное квантовое число ( & ell; ), равное двум (2).Следовательно, каждая орбиталь d имеет два угловых узла и возможные значения магнитного квантового числа m , равные -2, -1, 0, +1 и +2, что означает, что может быть пять орбиталей d . в любой из электронных оболочек от 3 n и далее. Большинство орбиталей d немного похоже на четырехлистный клевер, сделанный из двух пар гантелей, хотя одна выглядит как гантель, окруженная пончиком! Иллюстрация ниже должна помочь вам их визуализировать.


Электронные оболочки 3 n и выше содержат d орбиталей


Пять орбиталей d в электронной оболочке 3 n , вместе с одной орбиталью s и тремя орбиталями p , позволяют ей содержать в общей сложности восемнадцать (18) электронов (помните, что каждая орбиталь может содержать всего два электрона, независимо от сложности его геометрии).

Как и в случае орбиталей p , отдельные орбитали d идентифицируются в соответствии с их отношениями с произвольными осями x , y и z . Орбитали d yz , d xz и d xy лежат на плоскостях, образованных орбиталями y и z , x и z и x . и y осей соответственно, причем каждый лепесток образует угол в сорок пять градусов с каждой осью в этой плоскости.На орбите d x 2 y 2 одна пара лепестков лежит на оси x , а другая — на оси y .

Орбиталь d z 2 имеет несколько новую конфигурацию, в которой два лепестка лежат на оси z и окружены кольцевой орбиталью с центром на оси z .Пончик делится пополам по окружности плоскостью, образованной осями x и y .

Орбитали

f

Электронная оболочка 4 n содержит тридцать два (32) электрона. Первые восемнадцать из этих электронов занимают орбитали s , p и d . Последние четырнадцать электронов занимают f орбиталей.Фактически, орбитали f не начинают заполняться до тех пор, пока мы не дойдем до третьего элемента в пятом периоде периодической таблицы, иттрия (Y) — после того, как орбитали 5 p и 6 s имеют были заселены (потому что эти орбитали имеют более низкие уровни энергии, чем орбитали 4 f ).

Орбитали f имеют вторичное квантовое число ( & ell; ), равное три (3).Следовательно, каждая орбиталь f имеет три угловых узла, и возможные значения для магнитного квантового числа m равны -3, -2, -1, 0, +1, +2 и +3, что означает, что их может быть семь. f орбиталей в любой из электронных оболочек, начиная с 4 n и далее.

Семь орбиталей f в электронной оболочке 4 n вместе с одной орбиталью s , тремя орбиталями p и четырьмя орбиталями d позволяют ему содержать в общей сложности тридцать два o ( 32) электроны.Геометрия орбиталей f более сложна, чем любая из рассмотренных нами ранее орбиталей. Мы не будем пытаться описывать их здесь, но иллюстрация ниже должна помочь вам их визуализировать.


Электронные оболочки 4 n и выше содержат f орбиталей


Теоретически количество электронных оболочек может увеличиваться до бесконечности, и мы можем ожидать, что электроны будут заселять орбитали g , h и i (и далее).Однако, хотя периодическая таблица содержит элементы с частично заполненными электронными оболочками 5 n , 6 n и 7 n , ни один природный или искусственный элемент никогда не наблюдался, чтобы иметь более тридцати двух электронов в любом электронных оболочек. Поэтому с практической точки зрения наш интерес к орбиталям заканчивается орбиталью f .

Принцип aufbau

Принцип aufbau получил свое название от немецкого слова Aufbauprinzip , что в переводе означает принцип построения .Принцип, восходящий к 1920-м годам, был приписан Нильсу Бору и Вольфгангу Паули. В нем говорится следующее:

«Орбитали более низкой энергии сначала заполняются электронами, и только затем орбитали высокой энергии заполняются».

Этот принцип следует из естественной тенденции атомов принимать наиболее стабильную электронную конфигурацию. Мы уже видели, что электронная оболочка не обязательно должна быть полностью заполнена до того, как следующая электронная оболочка начнет заполняться электронами, потому что уровни энергии суборбиталей в двух электронных оболочках могут перекрываться.Итак, есть ли какие-то правила, которые мы можем применить здесь?

На самом деле существует относительно простое правило, которое можно использовать для определения — по крайней мере, в большинстве случаев — порядка, в котором электроны заполняют орбитали. Немецкий физик Эрвин Маделунг (1881-1972) разработал правило Маделунга (также известное под несколькими другими именами), согласно которому атомные орбитали заполняются в порядке возрастания значений: n + & ell; (т.е. сумма главного и вторичного квантовых чисел). Правило часто представляется графически с помощью следующей диаграммы:


Схематическое изображение правила Маделунга


Надеюсь, диаграмму относительно легко понять. Чтобы найти порядок, в котором подоболочки заполняются электронами, просто следуйте стрелкам сверху вниз и справа налево.Каждая запись дает главное квантовое число (которое идентифицирует электронную оболочку) и букву, которая представляет один из орбитальных типов ( s , p , d или f ). Обратите внимание: если две подоболочки имеют одинаковое значение n + & ell; , подоболочка с меньшим значением n заполнится первой. Вот полная последовательность:

1 с , 2 с , 2 с , 3 с , 3 с , 4 с , 3 с , 4 с , 5 с , 4 с , 5 p , 6 s , 4 p , 5 p , 6 p , 7 s , 5 p , 6 d , 7 p

Другой способ графического представления этой прогрессии показан ниже.Каждая подоболочка представлена ​​рядом прямоугольников, соответствующих количеству содержащихся в ней орбиталей определенного типа. Электронные оболочки показаны в строгом порядке слева направо, но последовательность, в которой заполняются подоболочки, является вертикальной (начиная с нижней части диаграммы).


На этой диаграмме показан (приблизительный) порядок заполнения электронных подоболочек.


Принцип aufbau, который был достигнут в результате обширных экспериментов, успешно предсказывает электронную конфигурацию всех элементов, кроме нескольких ( медь , хром и палладий являются примерами исключений из правила).Однако имейте в виду, что не всегда удается предсказать точный порядок , в котором заполнены подоболочки, и возникают аномалии с по .

Элемент , скандий (Sc) является хорошим примером. Скандий имеет атомный номер двадцать один (21), а значит, имеет двадцать один электрон в нейтральном (неионизированном) состоянии. Это первый из переходных металлов, относящийся к четвертому периоду периодической таблицы.Правило Маделунга успешно предсказывает окончательную конфигурацию электронов, которая выглядит следующим образом (обратите внимание, что цифры в верхнем индексе представляют количество электронов на каждой орбитали):

1 с 2 , 2 с 2 , 2 с 6 , 3 с 2 , 3 с 6 , 4 с 2 , 3 г 1

Однако в этом ли порядке заполняются подоболочки? Согласно полученным экспериментальным результатам, ответ отрицательный.Эти эксперименты показывают, что одиночный электрон, занимающий орбиталь 3 d , присутствует до того, как будет заполнена орбиталь 4 s .

Полное изучение того, почему возникает эта очевидная аномалия, выходит за рамки этой страницы. Однако обратите внимание, что важно знать о таких несоответствиях, особенно при рассмотрении того, что происходит во время ионизации. В случае скандия положительный ион Sc + образуется, когда электрон теряется с орбитали 4 s .В этом есть смысл, поскольку орбиталь 4 s — это последняя орбиталь, которую нужно заполнить.

Сокращение электронной конфигурации

Как мы видели выше для элемента скандий, электронная конфигурация любого элемента может быть записана с использованием обозначений, которые точно сообщают нам, что нам нужно знать с точки зрения как орбитального типа, так и количества электронов в каждой подоболочке.Для более легких элементов такая форма записи относительно проста. Например, электронная конфигурация для водорода записывается как 1 s 1 ; гелий записывается как 1 с 2 ; а литий записывается как 1 с 2 , 2 с 1 .

Однако, как мы видим из примера со скандием, при увеличении атомного номера сокращенная форма записи становится все более громоздкой.Когда мы переходим к более тяжелым элементам, становится ясно, что сокращенная версия обозначений, безусловно, сделает жизнь немного проще. К счастью, можно использовать сокращенную форму записи.

Наиболее часто используемой сокращенной версией электронной конфигурации, по-видимому, является конфигурация благородного газа , названная так потому, что в ней используется благородный газ, занимающий последний столбец периодической таблицы в строке, непосредственно предшествующей интересующему элементу.Обозначение по существу состоит из символа этого благородного газа, за которым следует стандартное обозначение для оставшейся подоболочки (ей).

Чтобы найти конфигурацию благородного газа для элемента, выполните следующие действия:

  1. Найдите элемент, конфигурацию которого вы хотите найти в периодической таблице.
  2. Определите, какой благородный газ использовать (это будет последний элемент в строке над элементом , для которого вы хотите найти конфигурацию).
  3. Напишите символ благородного газа в квадратных скобках, например [He], [Ne], [Ar] и т. Д. И отметьте как атомный номер благородного газа , так и номер строки , в которой он встречается.
  4. Определите, сколько электронов осталось (это будет атомный номер интересующего вас элемента за вычетом атомного номера благородного газа).
  5. Начните оставшуюся часть конфигурации с номера строки, в которой находится интересующий элемент, за которым следует орбитальный тип s (мы всегда начинаем с орбитального типа s ).
  6. Продолжайте записывать свою электронную конфигурацию в соответствии с диаграммой aufbau, которую мы видели ранее, пока не будут учтены все оставшиеся электроны.

Пример должен помочь прояснить процедуру. Найдем конфигурацию благородного газа для селена (Se). На изображении ниже показана соответствующая часть таблицы Менделеева.


Часть таблицы Менделеева с выделенными селеном и аргоном.


Селен находится в четвертой строке периодической таблицы, поэтому благородный газ, который мы должны использовать, — это аргон, который находится в последнем столбце строки три (3) и имеет атомный номер восемнадцать (18).Теперь мы можем начать нашу настройку, написав символ аргона в квадратных скобках, таким образом:

[Ar]

Атомный номер селена тридцать четыре (34). Вычитание атомного номера аргона дает нам в общей сложности шестнадцать (16) электронов, которые все еще необходимо учесть. Все идет нормально. Мы знаем, что селен находится в четвертой строке, поэтому можем написать:

[Ar] 4 с .. .

Нам еще нужно учесть шестнадцать электронов. На подоболочку 4 s приходится два из них. Вот последовательность, в которой подоболочки (начиная с 4 s и далее) заполняются по принципу aufbau:

4 с , 3 с , 4 с , 5 с , 4 с , 5 с , 6 с , 4 с , 5 с , 6 с , 7 с , 5 с , 6 с , 7 с

Полная подоболочка размером 3 d добавит к общему количеству на десять (10) электронов больше, оставив только четыре электрона для поиска.Поскольку подоболочка 4 p может содержать в общей сложности шесть электронов, нам не нужно искать дальше этого. Наша окончательная (сокращенная) конфигурация выглядит так:

[Ar] 4 с 2 , 3 d 10 , 4 p 4


Оболочки и подоболочки — A-Level Chemistry

Снаряды

Электроны вращаются вокруг ядра атома на разных расстояниях, называемых оболочками.

У каждой оболочки свой уровень энергии, который увеличивается по мере удаления от ядра.

Каждому энергетическому уровню дается число, называемое главным квантовым числом n. Ближайшая оболочка имеет значение n = 1. Следующая оболочка имеет значение n = 2 и т. Д.

Максимальное возможное количество электронов на первых четырех энергетических уровнях:

n = Ракушка Максимальное количество электронов
1 1-й Shell 2
2 2-й корпус 8
3 3-й корпус 18
4 4-й корпус 32

Используя вышеизложенное, вы можете определить максимальное количество электронов, которое может занимать оболочку, равно 2n 2 .

Электроны помещаются в доступные оболочки, начиная с самого низкого уровня энергии. Каждая оболочка должна быть заполнена до того, как начнет заполняться следующая. Эта модель не работает на оболочке n = 3, потому что каждая оболочка имеет подоболочки.

Подоболочки

Есть 4 подоболочки: s, p, d и f. Каждая подоболочка может содержать разное количество электронов.

Подоболочка Электроны
s 2
p 6
d 10
f 14

Число n определяет, сколько подоболочек составляет оболочку.Например, первая оболочка состоит из 1 подоболочки, s . Следовательно, он может содержать только 2 электрона.

Вторая оболочка состоит из двух подоболочек: s и p . Следовательно, он может содержать 2 + 6 = 8 электронов.

Полная таблица для первых четырех снарядов выглядит так:

Ракушка Подоболочка Общее количество электронов в оболочке
1-й Shell 2
2-й корпус 2с, 2п 2 + 6 = 8
3-й корпус 3с, 3п, 3д 2 + 6 + 10 = 18
4-й корпус 4с, 4п, 4д, 4ф 2 + 6 + 10 + 14 = 32

Число перед каждой подоболочкой указывает, к какой оболочке она принадлежит.

Например, у лития 3 электрона. 2 сначала заполнит 1-ю оболочку подоболочкой 1. Оставшийся электрон появится во второй оболочке в подоболочке 2s.

Вы можете написать полную электронную конфигурацию в виде подоболочек.

Возвращаясь к приведенному выше примеру, Литий равен 1s 2 2s 1 (1s имеет 2 электрона, 2s имеет 1 электрон).

Подобные электронные конфигурации внутри группы Периодической таблицы могут быть подчеркнуты с помощью более простого представления в терминах предыдущего благородного газа.

Литий 1s 2 2s 1 можно упростить до [He] 2s 1 , поскольку гелий (He) имеет электронную конфигурацию 1s 2 .

Примечание: подоболочки имеют разные уровни энергии, что может сбивать с толку порядок их заполнения.

Подоболочки и периодическая таблица

Элементы сгруппированы в блоки, которые относятся к подоболочке, содержащей электрон с наивысшей энергией.

Например, любой элемент в строке 3d будет иметь электрон наивысшей энергии в субоболочке d оболочки 3 rd , тогда как элемент в строке 4d будет иметь электрон наивысшей энергии в субоболочке d снаряда 4 -го .

1.3: Атомная структура — электронные конфигурации

Цель

После завершения этого раздела вы сможете записать электронную конфигурацию в основном состоянии для каждого из элементов до атомного номера 36 включительно.

Ключевые термины

Убедитесь, что вы можете определить и использовать в контексте следующие ключевые термины.

  • Основная электронная конфигурация
  • Правило Хунда
  • Принцип исключения Паули
  • принцип aufbau

Электронная конфигурация атома представляет собой расположение электронов, распределенных между орбитальными оболочками и подоболочками. Обычно электронная конфигурация используется для описания орбиталей атома в его основном состоянии, но ее также можно использовать для представления атома, который ионизировался в катион или анион, компенсируя потерю или усиление электронов в их последующих орбитали.Многие физические и химические свойства элементов можно соотнести с их уникальными электронными конфигурациями. Валентные электроны, электроны во внешней оболочке, являются определяющим фактором уникального химического состава элемента.

Электронные конфигурации

Электронная конфигурация атома представляет собой расположение электронов, распределенных между орбитальными оболочками и подоболочками. Обычно электронная конфигурация используется для описания орбиталей атома в его основном состоянии, но ее также можно использовать для представления атома, который ионизировался в катион или анион, компенсируя потерю или усиление электронов в их последующих орбитали.Многие физические и химические свойства элементов можно соотнести с их уникальными электронными конфигурациями. Валентные электроны, электроны во внешней оболочке, являются определяющим фактором уникального химического состава элемента.

Прежде чем отнести электроны атома к орбиталям, необходимо ознакомиться с основными концепциями электронных конфигураций. Каждый элемент периодической таблицы состоит из атомов, состоящих из протонов, нейтронов и электронов.Электроны обладают отрицательным зарядом и находятся вокруг ядра атома на электронных орбиталях, определяемых как объем пространства, в котором электрон может быть найден с вероятностью 95%. Четыре разных типа орбиталей (s, p, d и f) имеют разные формы, и одна орбиталь может содержать максимум два электрона. Орбитали p, d и f имеют разные подуровни, поэтому могут содержать больше электронов.

Как уже говорилось, электронная конфигурация каждого элемента уникальна в зависимости от его положения в периодической таблице.Уровень энергии определяется периодом, а количество электронов определяется атомным номером элемента. Орбитали на разных энергетических уровнях похожи друг на друга, но занимают разные области в космосе. Обе орбитали 1s и 2s имеют характеристики s-орбитали (радиальные узлы, вероятности сферического объема, могут содержать только два электрона и т. Д.), Но, поскольку они находятся на разных энергетических уровнях, они занимают разные пространства вокруг ядра. Каждая орбиталь может быть представлена ​​определенными блоками в периодической таблице.S-блок — это область щелочных металлов, включая гелий (группы 1 и 2), d-блок — это переходные металлы (группы с 3 по 12), p-блок — это основные элементы группы из групп с 13 по 18, и f-блок представляют собой ряды лантаноидов и актинидов.

Рисунок \ (\ PageIndex {1} \) Периодическая таблица

Использование периодической таблицы для определения электронной конфигурации атомов является ключевым моментом, но также помните, что существуют определенные правила, которым нужно следовать при назначении электронов на разные орбитали.Таблица Менделеева — невероятно полезный инструмент для написания электронных конфигураций. Для получения дополнительной информации о том, как связаны электронные конфигурации и периодическая таблица, посетите модуль «Подключение электронов к периодической таблице».

Правила назначения электронных орбиталей

Принцип исключения Паули

Принцип исключения Паули гласит, что никакие два электрона не могут иметь одинаковые четыре квантовых числа. Первые три (n, l и m l ) могут быть одинаковыми, но четвертое квантовое число должно быть другим.Одна орбиталь может содержать максимум два электрона, из которых должны, иметь противоположные спины; иначе у них были бы те же четыре квантовых числа, что запрещено. Один электрон вращается вверх (m s = +1/2), а другой вращается вниз (m s = -1/2). Это говорит нам о том, что каждая подоболочка имеет вдвое больше электронов на орбиталь. Подоболочка s имеет 1 орбиталь, которая может содержать до 2 электронов, подоболочка p имеет 3 орбитали, которые могут удерживать до 6 электронов, подоболочка d имеет 5 орбиталей, которые содержат до 10 электронов, а подоболочка f имеет 7 орбиталей с 14 электроны.

Пример 1: Водород и гелий

Первые три квантовых числа электрона: n = 1, l = 0, m l = 0. Им могут соответствовать только два электрона, которые будут либо m s = -1/2, либо m s = +1/2. Как мы уже знаем из наших исследований квантовых чисел и электронных орбиталей, мы можем сделать вывод, что эти четыре квантовых числа относятся к подоболочке единиц. Если дано только одно из значений m s , то у нас будет 1s 1 (обозначает водород), если даны оба значения, то получится 1s 2 (обозначает гелий).Визуально это будет представлено как:

Как показано, подоболочка 1s может содержать только два электрона, и, когда они заполнены, электроны имеют противоположные спины.

Правило Хунда

При назначении электронов на орбиталях каждый электрон сначала заполнит все орбитали одинаковой энергией (также называемой вырожденной), а затем спаривается с другим электроном на наполовину заполненной орбитали. Атомы в основном состоянии имеют как можно больше неспаренных электронов. Визуализируя эти процессы, подумайте о том, как электроны проявляют то же поведение, что и те же полюса магнита, если бы они вступили в контакт; поскольку отрицательно заряженные электроны заполняют орбитали, они сначала пытаются уйти как можно дальше друг от друга, прежде чем им придется образовывать пары.

Пример 2: Кислород и азот

Если мы посмотрим на правильную электронную конфигурацию атома азота (Z = 7), очень важного элемента в биологии растений: 1s 2 2s 2 2p 3

Мы можем ясно видеть, что p-орбитали заполнены наполовину, поскольку есть три электрона и три p-орбитали. Это связано с тем, что Правило Хунда гласит, что три электрона в подоболочке 2p сначала заполнят все пустые орбитали, а затем заполнят орбитали электронами в них.Если мы посмотрим на элемент после азота в тот же период, кислород (Z = 8), его электронная конфигурация будет: 1s 2 2s 2 2p 4 (для атома).

У кислорода на один электрон больше, чем у азота, и поскольку все орбитали заполнены наполовину, электрон должен образовать пары.

Занятие на орбите

Электроны заполняют орбитали таким образом, чтобы минимизировать энергию атома. Следовательно, электроны в атоме заполняют основные энергетические уровни в порядке увеличения энергии (электроны удаляются от ядра).Относительная энергия орбиталей показана на рисунке \ (\ PageIndex {2} \)

.

Рисунок \ (\ PageIndex {2} \) Относительная потенциальная энергия атомных орбиталей

Тогда порядок заполнения уровней:

1s, 2s, 2p, 3s, 3p, 4s, 3d, 4p, 5s, 4d, 5p, 6s, 4f, 5d, 6p, 7s, 5f, 6d и 7p

Общий порядок заполнения орбиталей показан на рисунке \ (\ PageIndex {3} \). Подоболочки, соответствующие каждому значению n , записываются слева направо на последовательных горизонтальных строках, где каждая строка представляет строку в периодической таблице.Порядок заполнения орбиталей обозначен диагональными линиями, идущими от верхнего правого угла до нижнего левого угла. Соответственно, орбиталь 4 s заполняется раньше орбитали 3 d из-за эффектов экранирования и проникновения. Следовательно, электронная конфигурация калия, которая начинается четвертый период, равна [Ar] 4 с 1 , а конфигурация кальция — [Ar] 4 с 2 . Пять 3 d орбиталей заполнены следующими 10 элементами, переходными металлами, за которыми следуют три 4 p орбиталей.Обратите внимание, что последним членом этой строки является криптон с благородным газом ( Z = 36), [Ar] 4 s 2 3 d 10 4 p 6 = [Kr], который заполнил 4 s , 3 d и 4 p орбиталей. Пятая строка периодической таблицы по существу такая же, как и четвертая, за исключением того, что орбитали 5 s , 4 d и 5 p заполняются последовательно.

Рисунок \ (\ PageIndex {3} \) : Предсказание порядка заполнения орбиталей многоэлектронными атомами.Если вы напишете подоболочки для каждого значения главного квантового числа в последовательных строках, наблюдаемый порядок, в котором они заполняются, будет обозначен серией диагональных линий, идущих от верхнего правого до нижнего левого.

Процесс Ауфбау

Aufbau происходит от немецкого слова «aufbauen», означающего «строить». При записи электронных конфигураций орбитали строятся от атома к атому. При записи электронной конфигурации атома орбитали заполняются в порядке увеличения атомного номера.Однако из этого правила есть некоторые исключения.

Пример 3: 3 рядных элемента

Следуя схеме в период от B (Z = 5) до Ne (Z = 10), количество электронов увеличивается, и подоболочки заполняются. В этом примере основное внимание уделяется подоболочке p, которая заполняется от бора до неона.

  • Конфигурация B (Z = 5): 1s 2 2s 2 2p 1
  • C (Z = 6) конфигурация: 1s 2 2s 2 2p 2
  • N (Z = 7) конфигурация: 1s 2 2s 2 2p 3
  • O (Z = 8) конфигурация: 1s 2 2s 2 2p 4
  • F (Z = 9) конфигурация: 1s 2 2s 2 2p 5
  • Ne (Z = 10) конфигурация: 1s 2 2s 2 2p 6

Число валентных электронов

Число валентных электронов элемента может быть определено группой периодической таблицы (вертикальный столбец), в которой этот элемент отнесен к категории.За исключением групп 3–12 (переходные металлы), цифра единиц номера группы указывает, сколько валентных электронов связано с нейтральным атомом элемента, указанного в этом конкретном столбце. Например, в группе 16 цифра единиц равна 6, а элементы в этой группе имеют 6 валентных электронов.

* Общий метод подсчета валентных электронов обычно не подходит для переходных металлов. Вместо этого используется модифицированный метод счета электронов d.

** За исключением гелия, который имеет только два валентных электрона.

Электронная конфигурация элемента — это расположение его электронов на его атомных орбиталях. Зная электронную конфигурацию элемента, мы можем предсказать и объяснить большую часть его химического состава.

Пример 1.3.1

Нарисуйте орбитальную диаграмму и используйте ее, чтобы вывести электронную конфигурацию фосфора, Z = 15. Какова его валентная электронная конфигурация?

Дано: атомный номер

Запрошено: орбитальная диаграмма и конфигурация валентных электронов для фосфора

Стратегия:

  1. Найдите ближайший благородный газ перед фосфором в периодической таблице.Затем вычтите его количество электронов из количества электронов в фосфоре, чтобы получить количество валентных электронов в фосфоре.
  2. Ссылаясь на рисунок 1.3.1, нарисуйте орбитальную диаграмму, чтобы представить эти валентные орбитали. Следуя правилу Хунда, поместите валентные электроны на доступные орбитали, начиная с орбитали с наименьшей энергией. Напишите конфигурацию электронов из вашей орбитальной диаграммы.
  3. Игнорируйте внутренние орбитали (те, которые соответствуют электронной конфигурации ближайшего благородного газа) и запишите конфигурацию валентных электронов для фосфора.

Раствор:

A Поскольку фосфор находится в третьей строке периодической таблицы, мы знаем, что он имеет замкнутую оболочку [Ne] с 10 электронами. Начнем с вычитания 10 электронов из 15 в фосфоре.

B Дополнительные пять электронов размещаются на следующих доступных орбиталях, которые, как показывает рисунок 1.2.5, это орбитали 3 s и 3 p :

Поскольку орбиталь 3 s имеет меньшую энергию, чем орбитали 3 p , мы сначала заполняем ее:

Правило Хунда говорит нам, что оставшиеся три электрона займут вырожденные 3 p орбитали отдельно, но со своими спинами выровненными:

Электронная конфигурация [Ne] 3 с 2 3 p 3 .

C Мы получаем конфигурацию валентных электронов, игнорируя внутренние орбитали, что для фосфора означает, что мы игнорируем закрытую оболочку [Ne]. Это дает конфигурацию валентных электронов 3 s 2 3 p 3 .

Упражнение 1.3.1

Нарисуйте орбитальную диаграмму и используйте ее, чтобы вывести электронную конфигурацию хлора, Z = 17. Какова его валентная электронная конфигурация?

Ответ
[Ne] 3 s 2 3 p 5 ; 3 с 2 3 с 5

Шестая строка периодической таблицы будет отличаться от двух предыдущих, потому что 4 f орбиталей, которые могут содержать 14 электронов, заполнены между 6 s и 5 d орбиталями.Элементы, содержащие в своей валентной оболочке 4 f орбиталей, являются лантаноидами. Когда орбитали 6 p , наконец, заполнены, мы достигли следующего (и последнего известного) благородного газа, радона ( Z = 86), [Xe] 6 s 2 4 f 14 5 d 10 6 p 6 = [Rn]. В последней строке 5 f орбиталей заполнены между 7 s и 6 d орбиталями, что дает 14 актинидных элементов.Поскольку большое количество протонов делает их ядра нестабильными, все актиниды радиоактивны.

Пример 1.3.2

Запишите электронную конфигурацию ртути ( Z = 80), указав все внутренние орбитали.

Дано: атомный номер

Запрошено: Полная электронная конфигурация

Стратегия:

Используя орбитальную диаграмму на рисунке 1.3.1 и периодическую таблицу в качестве руководства, заполните орбитали, пока не будут размещены все 80 электронов.

Раствор:

Размещая электроны на орбиталях в порядке, показанном на рисунке 1.3.1, и используя периодическую таблицу в качестве руководства, мы получаем

1 с 2 ряд 1 2 электрона
2 с 2 2 с 6 ряд 2 8 электронов
3 с 2 3 с 6 ряд 3 8 электронов
4 с 2 3 d 10 4 с 6 ряд 4 18 электронов
5 с 2 4 с 10 5 с 6 ряд 5 18 электронов
ряд 1–5 54 электронов
После заполнения первых пяти строк у нас все еще остается 80 — 54 = 26 электронов, которые нужно разместить.Согласно рисунку 1.3.1, нам нужно заполнить 6 s (2 электрона), 4 f (14 электронов) и 5 ​​ d (10 электронов) орбиталей. В результате получается электронная конфигурация ртути:

1 с 2 2 с 2 2 с 6 3 с 2 3 с 6 4 с 2 3 d 10 4 p 6 5 s 2 4 d 10 5 p 6 6 s 2 4 f 14 5 d 10 = Hg = [Xe] 6 s 2 4 f 14 5 d 10

с заполненной подоболочкой 5 d , a 6 s 2 4 f 14 5 d 10 конфигурация валентной оболочки и всего 80 электронов.(Вы всегда должны проверять, чтобы убедиться, что общее количество электронов равно атомному номеру.)

Сводка

На основе принципа Паули и знания орбитальных энергий, полученных с использованием водородоподобных орбиталей, можно построить периодическую таблицу, заполняя доступные орбитали, начиная с орбиталей с наименьшей энергией (принцип aufbau ), что дает поднимаются до определенного расположения электронов для каждого элемента (его электронная конфигурация ). Правило Хунда гласит, что расположение электронов с наименьшей энергией — это такое, при котором они располагаются на вырожденных орбиталях с параллельными спинами. Для химических целей наиболее важными электронами являются те, что находятся во внешней главной оболочке, валентных электронов .

Упражнения

1) Напишите электронную конфигурацию углерода и начертите орбитальную диаграмму.

2) Сколько валентных электронов имеют азот, углерод и хлор?

Решения

1) Электронная конфигурация углерода: 1s 2 2s 2 2p 6 , а орбитальная диаграмма изображена ниже.

2) Азот = 5, углерод = 4, хлор = 7.

вопросов

Q1.3.1

Приведите электронные конфигурации для Al, Br, Fe.

Решения

S1.3.1

Al = 1s 2 2s 2 2p 6 3s 2 3p 1

Br = 1s 2 2s 2 2p 6 3s 2 3p 6 4s 2 3d 10 4p 5

Fe = 1s 2 2s 2 2p 6 3s 2 3p 6 4s 2 3d 6

Авторы и авторство

электронных структур атомов

ЭЛЕКТРОННЫЕ КОНСТРУКЦИИ


 

На этой странице рассказывается, как писать электронные структуры атомов, используя обозначения s, p и d.Предполагается, что вы знаете о простых атомных орбиталях — по крайней мере, в том, как они названы, и их относительные энергии. Если вы хотите изучить электронные структуры простых одноатомных ионов (таких как Cl , Ca 2+ и Cr 3+ ), вы найдете ссылку внизу страницы.


Важно! Если вы еще не читали страницу об атомных орбиталях, вам следует перейти по этой ссылке, прежде чем идти дальше.


Электронное строение атомов

Связь орбитального заполнения с Периодической таблицей


Примечание: На некоторых экранах V для ванадия (элемент 23) может выглядеть немного как Y. Это не ошибка, а результат преобразования моей исходной диаграммы в изображение в формате gif более низкого качества для эффективного использования в Интернете.


Учебные планы Великобритании для 16-18-летних, как правило, останавливаются на криптоне, когда дело доходит до написания электронных структур, но возможно, что вас могут попросить структуры для элементов вплоть до бария.После бария вы должны беспокоиться о f-орбиталях, а также о s, p и d-орбиталях — и это проблема химии на более высоком уровне. Важно, чтобы вы просматривали прошлые экзаменационные работы, а также свою программу, чтобы вы могли оценить, насколько сложными могут быть вопросы.

На этой странице подробно рассматриваются элементы в сокращенной версии Периодической таблицы, приведенной выше, а затем показано, как вы могли бы разработать структуры некоторых более крупных атомов.


Важно! У вас должна быть копия программы и копии недавних экзаменационных работ.Если вы изучаете учебную программу в Великобритании и не получили ее, перейдите по этой ссылке, чтобы узнать, как их получить.


Первый период

Водород имеет свой единственный электрон на орбитали 1s — 1s 1 , а у гелия первый уровень полностью заполнен — ​​ 1s 2 .

Второй период

Теперь нам нужно начать заполнение второго уровня, а значит, начать второй период.Электрон лития переходит на 2s-орбиталь, потому что он имеет более низкую энергию, чем 2p-орбитали. Литий имеет электронную структуру 1s 2 2s 1 . Бериллий добавляет к этому же уровню второй электрон — 1s 2 2s 2 .

Теперь начинают заполняться 2p уровни. Все эти уровни имеют одинаковую энергию, поэтому электроны сначала входят поодиночке.

B 1s 2 2s 2 2p x 1
C 1s 2 2s 2 2s 900 2p y 1
N 1s 2 2s 2 2p x 1 2p y 1 2p y 1 2p

Примечание: Орбитали, на которых происходит что-то новое, выделены жирным шрифтом.Обычно вы не записываете их иначе, чем другие орбитали.


Следующие электроны, которые войдут внутрь, должны будут образовать пары с уже существующими.

2 2s 2 2p x 2 2p y 2 2p z 1
O 1s 2 2s 2 2p x 2 2p y 1 2p z 1
Ne 1s 2s 4 2 2 2 полюса y 2 2 полюса z 2

Вы можете видеть, что по мере увеличения числа электронов писать полные электронные структуры атомов становится все более утомительно.Есть два способа обойти это, и вы должны знать оба.

Ярлык 1: Все различные p-электроны можно объединить в одну группу. Например, фтор можно записать как 1s 2 2s 2 2p 5 , а неон — как 1s 2 2s 2 2p 6 .

Это то, что обычно происходит, если электроны находятся во внутреннем слое. Если электроны находятся на уровне связи (те, что снаружи атома), они иногда записываются сокращенно, иногда полностью.Не беспокойся об этом. Будьте готовы встретиться с любой версией, но если вас спросят об электронной структуре чего-либо на экзамене, запишите ее полностью, указав все орбитали p x , p y и p z на внешнем уровне по отдельности. .

Например, хотя мы еще не познакомились с электронной структурой хлора, вы могли бы записать ее как 1s 2 2s 2 2p 6 3s 2 3p x 2 3p y 2 3p z 1 .

Обратите внимание, что все 2p-электроны сгруппированы вместе, а 3p-электроны показаны полностью. Логика состоит в том, что 3p-электроны будут участвовать в связывании, потому что они находятся вне атома, тогда как 2p-электроны похоронены глубоко в атоме и на самом деле не представляют интереса.

Ярлык 2: Можно объединить всех внутренних электронов вместе, используя, например, символ [Ne]. В этом контексте [Ne] означает электронная структура неона — другими словами: 1s 2 2s 2 2p x 2 2p y 2 2p z 2 You не будет делать этого с гелием, потому что для записи [He] требуется больше времени, чем для 1s 2 .

На этом основании структура хлора будет записана [Ne] 3s 2 3p x 2 3p y 2 3p z 1 .

Третий период

У неона все орбитали второго уровня заполнены, поэтому после этого мы должны начать третий период с натрием. Схема заполнения теперь точно такая же, как и в предыдущий период, за исключением того, что теперь все происходит на 3-м уровне.

Например:


Примечание: Убедитесь, что вы можете это сделать. Закройте текст, а затем разработайте эти структуры для себя. Затем проделайте все остальное в этот период. Когда вы закончите, сравните свои ответы с соответствующими элементами предыдущего периода. Ваши ответы должны быть такими же, за исключением следующего уровня.


Начало четвертого периода

На данный момент не все трехуровневые орбитали заполнены — трехмерные уровни еще не использовались.Но если вы вернетесь к энергиям орбиталей, вы увидите, что следующая самая низкая энергетическая орбиталь — это 4s, так что она заполняется следующей.

2 Ca 2p 6 3s 2 3p 6 4s 2
K 1s 2 2s 2 2p 6 3s 2 3p 6 4s 1

Имеются убедительные доказательства этого в сходстве химии таких элементов, как натрий (1s 2 2s 2 2p 6 3s 1 ) и калий (1s 2 2s 2 2p 6 3s 2 3p 6 4s 1 )

Внешний электрон управляет их свойствами, и этот электрон находится на одной и той же орбитали в обоих элементах.Это было бы неверно, если бы внешний электрон в калии был 3d 1 .

s- и p-блочные элементы

Все элементы группы 1 Периодической таблицы имеют внешнюю электронную структуру ns 1 (где n — число от 2 до 7). Все элементы группы 2 имеют внешнюю электронную структуру ns 2 . Элементы в группах 1 и 2 описываются как элементы s-блока.

Элементы из группы 3 (группа бора) и благородных газов имеют внешние электроны на p-орбиталях.Затем они описываются как p-блочные элементы.


Примечание: Если вы используете текущую систему ИЮПАК (Международный союз теоретической и прикладной химии) для групповой нумерации, вы, вероятно, будете знать, что я называю Группой 3 как Группой 13. Мои причины не использовать систему ИЮПАК обсуждаются здесь. в разделе «Вопросы и комментарии».


d-образные элементы

Мы разрабатываем электронные структуры атомов, используя принцип Aufbau («наращивание»).Итак, мы получили кальций со структурой 1s 2 2s 2 2p 6 3s 2 3p 6 4s 2 .

Уровень 4s теперь заполнен, и структуры следующих атомов показывают, что электроны постепенно заполняют уровень 3d. Они известны как элементы d-блока.

После заполнения трехмерных орбиталей следующие электроны переходят на 4р-орбитали, как и следовало ожидать.

элементов d-блока — это элементы, в которых последний электрон, добавляемый к атому с использованием принципа Ауфбау, находится на d-орбитали.

Первая серия содержит элементы от скандия до цинка, которые на экзамене GCSE вы, вероятно, назвали переходными элементами или переходными металлами. Термины «элемент перехода» и «элемент d-блока» не имеют одинакового значения, но в данном контексте это не имеет значения.


Если вам интересно: Переходный элемент определяется как элемент, который имеет частично заполненных d-орбиталей либо в элементе, либо в любом из его соединений.Цинк (в правом конце d-блока) всегда имеет полностью полный 3-й уровень (3d 10 ) и поэтому не считается переходным элементом.

Некоторые учебные программы Великобритании используют более ограничительное определение, которое определяет переходный металл как металл, который имеет один или несколько стабильных ионов с частично заполненными d-орбиталями. Вам не нужно беспокоиться об этом, пока вы не изучите химию переходных металлов.



d-электронов почти всегда описываются как, например, d 5 или d 8 — и не записываются как отдельные орбитали.Помните, что существует пять d-орбиталей, и что электроны будут населять их поодиночке, насколько это возможно. До 5 электронов самостоятельно займут орбитали. После этого им придется разделиться на пары.

d 5 означает

d 8 означает

Обратите внимание, что все трехуровневые орбитали записываются вместе — 4s-электроны записываются в конце электронной структуры.

Sc 1s 2 2s 2 2p 6 3s 2 3p 6 3d 1 4s 2 9015 2s 2 2p 6 3s 2 3p 6 3d 2 4s 2
V 1s 2 2s 2 2s 2 3p 6 3d 3 4s 2
Cr 1s 2 2s 2 2p 6 3s 3 4с 1

Ой! Хром нарушает последовательность.В хроме электроны на 3d- и 4s-орбиталях перестраиваются так, что на каждой орбитали находится по одному электрону. Было бы удобно, если бы последовательность была аккуратной — но это не так!

А у цинка процесс заполнения d-орбиталей завершен.

Заполнение до конца периода 4

Следующие орбитали, которые будут использоваться, — это 4p, и они заполняются точно так же, как 2p или 3p. Мы вернулись к элементам p-блока от галлия до криптона.Бром, например, 1s 2 2s 2 2p 6 3s 2 3p 6 3d 10 4s 2 4p x 2 4p z 1 .


Полезное упражнение: Определите электронные структуры всех элементов от галлия до криптона. Вы можете проверить свои ответы, сравнив их с элементами, находящимися прямо над ними в Периодической таблице.Например, галлий будет иметь такое же расположение электронов внешнего уровня, что и бор или алюминий, за исключением того, что внешние электроны галлия будут находиться на 4-уровне.


Сводка

Запись электронной структуры элемента от водорода до криптона

  • Используйте Периодическую таблицу, чтобы найти атомный номер и, следовательно, количество электронов.

  • Заполните орбитали в порядке 1s, 2s, 2p, 3s, 3p, 4s, 3d, 4p — пока у вас не закончатся электроны.3-й — неудобный — запомните это особо. Заполните p- и d-орбитали по отдельности, насколько это возможно, прежде чем объединять электроны в пары.

  • Помните, что хром и медь имеют электронные структуры, которые нарушают структуру в первом ряду d-блока.


 

Запись электронной структуры больших элементов s- или p-блока


Примечание: Мы сознательно исключаем элементы d-блока, кроме первой строки, которую мы уже подробно рассмотрели.Картина неудобных структур отличается в других рядах. Это проблема для степени.


Сначала определите количество внешних электронов. Вполне вероятно, что это все, что вас все равно попросят сделать.

Число внешних электронов такое же, как и номер группы. (Благородные газы здесь представляют собой небольшую проблему, потому что их обычно называют группой 0, а не группой 8. Гелий имеет 2 внешних электрона, а остальные — 8.) Все элементы в группе 3, например, имеют 3 электрона на внешнем уровне. При необходимости поместите эти электроны на s- и p-орбитали. Орбитали какого уровня? Подсчитайте периоды в Периодической таблице (не забывая тот, в котором есть H и He).

Йод находится в группе 7 и, следовательно, имеет 7 внешних электронов. Он находится в пятом периоде, поэтому его электроны будут на 5s и 5p орбиталях. Йод имеет внешнюю структуру 5s 2 5p x 2 5p y 2 5p z 1 .

А как насчет внутренних электронов, если их тоже нужно вычислить? Уровни 1, 2 и 3 будут заполнены, как и уровни 4, 4 и 4. Уровни 4f не заполняются до тех пор, пока вас не спросят о чем-либо на A’level. Просто забудьте о них! Это дает полную структуру: 1s 2 2s 2 2p 6 3s 2 3p 6 3d 10 4s 2 4p 6 4d 10 5s x 5p 2 5p y 2 5p z 1 .

Когда вы закончите, посчитайте все электроны, чтобы убедиться, что они совпадают с атомным номером. Не забудьте сделать эту проверку — легко пропустить выход на орбиту, когда все усложняется.

Барий находится в группе 2 и, следовательно, имеет 2 внешних электрона. Это в шестом периоде. Барий имеет внешнюю структуру 6s 2 .

Включая все внутренние уровни: 1s 2 2s 2 2p 6 3s 2 3p 6 3d 10 4s 2 4p 6 4d 10 6s 9014p3 2 6с 2 .

Было бы легко включить 5d 10 по ошибке, но уровень d всегда заполняет после следующего уровня s, поэтому 5d заполняется через 6 секунд так же, как 3d заполняется через 4 секунды. Если вы посчитаете количество электронов, вы легко заметите эту ошибку, потому что их будет на 10 больше.


Примечание: Не беспокойтесь об этих сложных конструкциях. Вам нужно знать, как их решать в принципе, но ваши экзаменаторы с гораздо большей вероятностью попросят у вас что-нибудь простое, например серу или железо.



 

Куда бы вы сейчас хотели отправиться?

К разработке электронных структур ионов. . .

В меню атомарных свойств. . .

В меню атомной структуры и связывания. . .

В главное меню. . .


 

© Джим Кларк 2000 (последнее изменение — октябрь 2012 г.)

Что такое валентные электроны и как их найти? Где они расположены?

Валентные электроны — это те электроны, которые находятся во внешней оболочке, окружающей атомное ядро.Валентные электроны имеют решающее значение, потому что они дают глубокое понимание химических свойств элемента: является ли он электроотрицательным или электроположительным по природе, или они указывают порядок связи в химическом соединении — количество связей, которые могут быть образованы между двумя атомами.

Поскольку ковалентные связи образуются за счет разделения электронов в конечной оболочке, число указывает, сколько связей может быть образовано.

Что такое валентные электроны?

Валентные электроны — это электроны, расположенные на внешней оболочке атома.Другими словами, это электроны, которые могут быть получены или потеряны в ходе химической реакции.

Где валентные электроны?

Независимо от типа химической связи между атомами, будь то ионная, ковалентная или металлическая связь, изменения в атомной структуре ограничиваются электронами во внешней оболочке, то есть валентными электронами.

Самый простой метод — это сослаться на атомную конфигурацию элемента и просто подсчитать электроны в самой внешней оболочке.Однако это будет чрезвычайно трудоемкая задача, поскольку нам, возможно, придется рыться в учебниках, чтобы найти конфигурации, которых мы не знаем.

Однако не стоит беспокоиться, так как существует гораздо более простой способ определения этого желанного числа. Это более общий подход, требующий всего лишь одного небольшого великолепного прямоугольного листа бумаги — таблицы Менделеева.

Чтобы определить количество валентных электронов элемента, нам нужно только обратиться к периодической таблице и найти положение элемента в ней.

Валентные электроны и таблица Менделеева

Периодическая таблица Менделеева представляет собой четкое расположение всех элементов, открытых на данный момент. Элементы расположены слева направо в порядке возрастания их атомных номеров или количества протонов или электронов, которые они содержат.

Элементы делятся на четыре категории: элементы основной группы, переходные элементы, лантаноиды и актиниды. Последние два также называются внутренними переходными элементами .

Таблица содержит всего 18 столбцов, формально известных как группы , , а также строки, формально известные как периоды . В подтаблице вверху 7 строк, а внизу — 2 строки. Переходные элементы образуют мост или закрепляют переход между элементами 2-й и 13-й групп.

Как найти валентные электроны?

Когда мы спускаемся по группе, количество валентных электронов остается прежним, хотя количество оболочек увеличивается.

В то время как валентные электроны за период постепенно поднимаются на единицу, количество оболочек остается неизменным. Номер периода (номер строки, чтобы напомнить вам), в котором можно найти элемент, указывает количество оболочек, окружающих его ядро.

Итак, какое значение имеет номер группы?

Валентные электроны элементов, отличных от переходных — элементы основной группы

В то время как номер периода указывает количество оболочек, номер группы указывает количество валентных электронов во внешней оболочке.В частности, число в разряде единиц. Однако это верно только для основных элементов группы — элементов, населяющих группы 1-2 и 13-18.

Правило неприменимо к элементам перехода и внутренним элементам перехода (мы рассмотрим эту причину через минуту). Например, натрий (Na) находится в периоде 3, группе 1, что означает, что он имеет 3 оболочки и один электрон в валентной оболочке.

Или вы можете рассматривать хлор в группе 17. Соответственно, чтобы определить его валентные электроны, мы должны искать только число вместо единиц: 7.Как и ожидалось, это именно количество электронов в его валентной оболочке.

Этот метод простого обращения к периодической таблице и определения соответствующего номера группы устранил хлопоты и сложности, которые когда-то сопровождали трудный поиск индивидуальных атомных конфигураций.

А как насчет валентных электронов элементов между ними? Конечно, мы не должны забывать о лантаноидах и актинидах…

Краткое объяснение того, как оболочки заполняются электронами

Переходные элементы мало чем отличаются от металлов, которые идут плечом к плечу в элементах основной группы.Они очень похожи на металлы: они пластичны, пластичны и могут проводить как тепло, так и электричество. Тот факт, что два лучших проводника — медь (Cu) и алюминий (Al) — являются переходными металлами, показывает, насколько их свойства перекрываются.

Однако они не дублируют результаты, полученные нами с помощью вышеуказанного метода. Мы не можем подсчитать их валентные электроны, просто ссылаясь на номер их группы.

Чтобы понять это исключение, нам нужно понять, как электроны занимают оболочки в любом элементе .

Однако сначала мы должны отучиться от школьного метода заполнения оболочек вокруг атомного ядра: помните 2..8..8..18 и так далее? Что ж, есть причина, по которой мы распределяем электроны именно таким образом.

Аналогия с солнечной системой, которая описывает упорядочение электронов вокруг атома, совершенно неверна. Его следует немедленно устранить, но поскольку он снимает трудности, связанные с представлением модели закона ual , учебники для старших классов в основном полагаются на это элементарное объяснение.

Электроны не занимают твердые оболочки вокруг своего ядра. Фактически, их положение вокруг ядра весьма неопределенно. Они могут занимать только определенные энергетические уровни вокруг ядра. Скорее всего, их там и можно найти. Технически эти уровни называются квантовыми состояниями и обозначаются так называемыми квантовыми числами n .

Теперь следующее предложение может показаться лицемерным, но квантовые числа можно рассматривать как старые добрые оболочки, но теперь с подоболочками, которые технически известны как орбитали (s, p, d, f).Несмотря на это упрощение, он вполне подходит для ускоренного курса, подобного этому.

Существует правило, ограничивающее количество электронов, которые может вместить суб-оболочка: s-2, p-6, d-10 и f-14. Если этого было недостаточно, добавляя к бреду, оболочки можно заполнять только в определенном порядке, указанном ниже. Назовем это правилом правилом .

Электроны должны заполняться только слева направо именно в этом порядке.

Если бы мы бессознательно распределили электроны относительно того, как выстроены суб-оболочки, как показано на рисунке выше, кальций (Ca) с атомным номером 20 имел бы конфигурацию 2,8,10 (2, 2+ 6, 2 + 6 + 2).Любой школьный учебник химии скажет вам, что это неверно, поскольку точная конфигурация — 2,8,8,2.

Однако, поскольку мы должны соблюдать правило , мы наблюдаем, что 4 должны быть заполнены до 3d, , так что теперь 8 в 3-й оболочке и 2 в 4-й, что составляет конфигурацию: 2,8,8 , 2. Вуаля! Как весело воскликнул бы Ричард Фейнман: «Удовольствие узнавать все! К сожалению, радость наполовину прожита — причина , самого правила , эта кажущаяся абсурдность, выходит за рамки данной статьи.

Хорошо, теперь, когда мы знаем, как заполняются оболочки, мы можем пойти дальше и найти количество валентных электронов в переходных элементах.

Валентные электроны лантаноидов и актинидов (переходные и внутренние переходные элементы)

Рассмотрим скандий (Sc) с его атомным номером 21. Заполняя электроны в соответствии с нашим правилом , , мы видим, что 21-й электрон занимает 3d-суб- оболочка. Однако, поскольку ранее заполненная 4-я оболочка (4s) имеет 2 электрона и, по-видимому, является самой внешней оболочкой, количество валентных электронов равно 2.

Аналогично, каждый переходный элемент в 4-м периоде должен иметь 2 валентных электрона. Причина в том, что хотя 3d заполняется раньше, чем 4s, два электрона, расположенные в 4-й оболочке, являются обитателями самой внешней оболочки и по праву заслуживают обозначения валентных электронов.

Фактически, это верно для переходных элементов в каждый период. Рассмотрим Золото (Au), находящееся в 6 периоде (строке) и 11 группе (столбце). В процессе наполнения его раковин можно понять, что после начинки 5d следует начинка 6s.А поскольку 6-я оболочка расположена выше 5-й, количество валентных электронов составляет … * барабанная дробь * … 2!

Тем не менее, именно так электроны в идеале и выстроились бы в линию. Энергетические различия между этими оболочками ничтожны, и электроны (или Природа, если на то пошло) жаждут стабильности больше всего на свете. Электрон с радостью совершит прыжок на соседнюю оболочку с относительно эквивалентной энергией, чтобы достичь более стабильной конфигурации.

Хорошим примером является непостоянная конфигурация атома меди (Cu).

Загадочный случай валентных электронов меди

Медь имеет 29 электронов, поэтому самые задние электроны выстраиваются в линию как… 4s2-3d9. Для меди конфигурация немного тревожит — более устойчивая конфигурация будет иметь 10 электронов в 3d-оболочке, и это именно то, что мы наблюдаем!

Так как энергии оболочек сравнимы, электрон 4s совершает скачок в 3d, чтобы получить стабильную конфигурацию. Количество валентных электронов теперь 1!

Ряд элементов среди переходных элементов демонстрируют эту странность, которая также наблюдается на внутренних переходных элементах из-за сравнимых уровней энергии оболочек f, d и s.

Таким образом, можно сказать, что количество валентных электронов для переходных элементов и внутренних переходных элементов изменяется непредсказуемым образом.

Хотя количество валентных электронов для переходных элементов все еще можно предсказать — и большинство из них в конечном итоге равно 2 — такое предсказание для внутренних переходных элементов невозможно имитировать.

Статьи по теме

Статьи по теме

Прихотливое поведение их валентных электронов, бесконечно дрожащих и подпрыгивающих в нерешительности, отрицает любые попытки получить уникальную стабильную конфигурацию, что делает предсказать количество валентных электронов практически невозможным!

Все, что вам нужно знать — Shemmassian Academic Consulting

Часть 3: Периодическая таблица

a) Группы

Понимание тенденций в периодической таблице может быть чрезвычайно полезным на MCAT.На экзамене вам будет предоставлена ​​таблица Менделеева, и вас могут попросить использовать ее в качестве инструмента, чтобы применить свои знания о периодических тенденциях для сравнения свойств элементов.

Одна из самых основных и важных тенденций — это то, что между группами . Таблица Менделеева состоит из 18 вертикальных столбцов, каждая из которых называется группой. Каждая группа в периодической таблице содержит элементы с разным количеством валентных электронов. Элементы группы 2 имеют 2 валентных электрона, элементы группы 13 имеют 3 валентных электрона и так далее.

Элементы в группе имеют схожее атомное расположение и, следовательно, свойства. Например, каждый из элементов группы 18 (благородные газы) имеет полную внешнюю оболочку из валентных электронов. Это означает, что благородные газы не обладают реакционной способностью, и эту тенденцию мы видим, когда спускаемся вниз по группе.

Элементы группы 1, щелочных металлов , обладают только одним валентным электроном и обладают высокой реакционной способностью. Элементы группы 2, щелочноземельных металлов , также весьма реакционноспособны. Как правило, элементы в крайнем левом углу таблицы Менделеева обладают высокой реакционной способностью по сравнению с элементами в центре таблицы Менделеева.

Галогены находятся в группе 17 периодической таблицы. Все эти элементы находятся в двухатомном состоянии , что означает, что в одной молекуле есть два атома. Поскольку этим элементам требуется только один дополнительный валентный электрон для выполнения правила октета, они обладают высокой реакционной способностью и часто встречаются в солевых соединениях.

халькогенов расположены в группе 16 периодической таблицы. Их также называют кислородной группой , , поскольку эта группа содержит элемент кислород.

переходных металлов находятся в группах 3-12 периодической таблицы. Их также можно называть элементами d-блока , , поскольку их валентная оболочка является d подоболочкой.

репрезентативных элементов находятся во всех других группах периодической таблицы. В результате они включают в себя элементы p-блока s- и .

б) Периодические тренды

Периодическая таблица тщательно организована таким образом, что документирует несколько тенденций или изменений свойств каждого элемента.Важно понимать каждую из этих периодических тенденций и уметь применять их в тестовый день.

Электроотрицательность относится к силе притяжения, которую ядро ​​атома оказывает для притяжения электрона. Напомним, что элементы наиболее стабильны, когда внешняя электронная оболочка заполнена. Таким образом, элементы с почти полной валентной оболочкой имеют более высокую электроотрицательность по сравнению с элементами с одним или двумя электронами в валентной оболочке. С энергетической точки зрения, атомы с почти полной электронной оболочкой могут обрести стабильность, притягивая последний электрон, в то время как элементы с небольшим количеством валентных электронов могут легче всего получить стабильность, выпуская эти электроны.Таким образом, электроотрицательность элементов возрастает вверх и вправо от таблицы Менделеева. Фтор (F), галоген, сильно электроотрицателен, тогда как барий (Ba), щелочноземельный металл, не очень электроотрицателен. Исключение, конечно, составляют благородные газы. Благодаря своей высокой стабильности благородные газы имеют почти нулевую электроотрицательность.

Энергия ионизации означает количество энергии, необходимое для удаления электрона из валентной оболочки атома. Возьмем два атома на противоположных сторонах таблицы Менделеева: калий (K) и хлор (Cl).Между тем, хлор имеет более высокую энергию ионизации валентной оболочки по сравнению с калием. Вы можете подумать, почему это могло быть?

Валентная оболочка нейтрального атома хлора заполнена 7 электронами. Валентная оболочка почти заполнена. С точки зрения стабильности чрезвычайно выгодно получить последний электрон и заполнить валентную оболочку. С другой стороны, у калия есть один неподеленный электрон в валентной оболочке. С точки зрения стабильности чрезвычайно выгодно потерять этот электрон — таким образом, превратив валентную оболочку в полную электронную оболочку одного более низкого периода.Таким образом, энергия ионизации увеличивается вверх и вправо от таблицы Менделеева.

Обратите внимание, что вторая энергия ионизации атома значительно выше, чем первая энергия ионизации. Это связано с тем, что для ионизации заряженной частицы требуется больше энергии по сравнению с ионизацией нейтральной частицы.

Сродство к электрону означает количество энергии, выделяемой электроном, когда он притягивает электрон. Сродство к электрону тесно связано с энергией ионизации и электроотрицательностью.Сродство к электрону увеличивается вверх и вправо от таблицы Менделеева. Это связано с тем, что силы притяжения между ядром и валентными электронами увеличиваются по мере уменьшения расстояния до валентной оболочки, а добавление протонов к ядру при сохранении этого расстояния постоянным будет увеличивать электростатическое притяжение.

Еще одна важная тенденция, которую необходимо понять, — это атомный размер. Атомный радиус уменьшается вверх и вправо от таблицы Менделеева.В первую очередь это связано с явлением, известным как эффективный ядерный заряд.

Добавить комментарий

Ваш адрес email не будет опубликован. Обязательные поля помечены *